You are on page 1of 53

Table of Contents

Assessment Activities………………………………………………………..…………….49-50
General Mathematics Project…………………………………………..…….…….………….…51-52
MODULE 9
MODULE 13
Chapter 2
General Mathematics 2nd Quarter Examination
2.1 Business Math
2.1.1 Simple Interest…………..…………………………………………1-2
2.1.2 Maturity (Future) Value……..……………………………..….3
2.1.3 Compound Interest…………………………………………….…3-5
2.1.4 Compounding More Than Once a Year………………….5-6
2.1.5 Simple Annuity……………………………………………………..7-9
2.1.6 General Annuity……………………………………………………9-11
2.1.7 Cash Flow and Fair Market Value………………………….11-12
2.1.8 Deferred Annuity………………………………………………….13
Assessment Activities…………………………………………………………..15-16
MODULE 10
2.2 Stock, Bonds, and Loans
2.2.1 Stock and Bonds…………………………………………………..17-20
2.2.2 Market Indices for Stocks and Bonds……………………20-22
2.2.3 Business and Consumer Loans……………………………..22-23
2.2.4 Amortization and Mortgage…………………………………23-24
Assessment Activities………………………………………………………….25-27
MODULE 11
Chapter 3
3.1 Logic
3.1.1 Propositions………..……………………………………………….28-29
3.1.2 Logical Operators…………………………………………………30-33
3.1.3 Truth Tables…………………………………………………………33-34
3.1.4 Logical Equivalence and Logical Propositions……….34-37
Assessment Activities………………………………………………………….38-40
MODULE 12
3.1.5 Valid Arguments and Fallacies……………………………...41-45
3.1.6 Valid and Sound Arguments……..…………………….…….45
3.1.7 Methods of Proof and Disproof………………………..…..45-47
ASIAN INSTITUTE OF COMPUTER STUDIES (AICS)

CHAPTER 2.1 BUSINESS MATH 


KEY QUESTIONS
What is the difference
2ND QUARTER

At the end of this module, you are expected to: between simple and
WEEK _____
MODULE 9

 Illustrate and distinguish simple and compound interests, compound interest?


 Compute interest, maturity value, future value, and present value in simple and compound interest  Does computing for the
environment, fair market value results
 Differentiate simple and general annuities: its future and present values, and differently depending on
 Calculate the fair market value of a cash flow stream that includes an annuity. the focal date chosen?
 
Definition of Terms Maturity Value or Future Value (F) – amount after t years that the lender receives
from the borrower on the maturity date.
Lender or Creditor – person (or institution) who invests the money or makes the
fund available. 2.1.1 Simple Interest
Borrower or Debtor – person (or institution) who owes the money or avails of the
funds from the lender. Annual Simple Interest
𝐼𝑠 = 𝑃𝑟𝑡
Origin or Loan Date – date on which the money is received by the borrower. Where,
𝐼𝑠 = simple interest
Repayment Date or Maturity Date – date on which the money borrowed, or loan
𝑃 = principal, or amount invested or borrowed
is to be completely repaid.
𝑟 = simple interest date
Time or Term (t) – amount of time in years the money is borrowed or invested; 𝑡 = term or time in years
length of time between the origin and maturity dates.
Example 1. A bank offers 0.25% annual simple interest for a particular deposit. How
Principal (P) – amount of money borrowed or invested on the origin date. much interest will be earned if 1 million pesos is deposited in this savings account
Rate (r) – annual rate, usually in percent, charged by the lender, or rate of increase for 1 year?
of the investment. Given.
Interest (I) – amount paid or earned for the use of money. 𝑃 = 1,000,000
𝑟 = 0.25% = 0.0025
Simple Interest (𝑰𝒔 ) – interest that is computed on the principal and then added 𝑡 = 1 year
to it.
Find 𝑰𝒔
Compound Interest (𝑰𝒄 ) – interest is computed on the principal and also on the Solution. 𝐼𝑠 = 𝑃𝑟𝑡
accumulated past interests. 𝐼𝑠 = (1,000,000)(0.0025)(1)

1
ASIAN INSTITUTE OF COMPUTER STUDIES (AICS)

𝐼𝑠 = 2,500 Principal (𝑃) Rate (𝑟) Time (𝑡) Interest (𝐼𝑠 )


(a) 3.5% 3 1,800
Answer. The interest earned is P2,500.
36,000 (b) 1.5 4,860
150,000 0.5% (c) 3,000
Example 2. Find the interest on a used car loan of P5,000 at a rate of 16% for a 500,000 12.5% 10 (d)
period of 3 years.
Given. Solution.
𝑃 = 5,000 (a) The unknown principal can be obtained by
𝑟 = 16% = 0.16 𝐼𝑠
𝑡 = 3 years 𝑃=
𝑟𝑡
Find 𝑰𝒔 1,800
𝑃=
Solution. 𝐼𝑠 = 𝑃𝑟𝑡 (0.035)(3)
𝐼𝑠 = (5,000)(0.16)(3) 𝑃 = 17,142.86
𝐼𝑠 = 2,400
(b) The unknown rate can be obtained by
Answer. The interest earned is P 2,400.
𝐼𝑠
𝑟=
𝑃𝑡
Example 3. How much interest is charged when P50,000 is borrowed for 9 months 4,860
at an annual interest rate of 10%? 𝑟=
(36,000)(1.5)
Given. 𝑟 = 0.09 = 9%
Reminder. When the term is
𝑃 = 50,000 (c) The unknown time can be obtained by
expressed in months (𝑀), it
𝑟 = 10% = 0.10 𝐼𝑠
9 should be converted in years 𝑡=
𝑡 = 12 year= 0.75 year 𝑀 𝑃𝑟
by 𝑡 = 12. 3,000
𝑡=
Find 𝑰𝒔 (150,000)(0.005)
Solution. 𝐼𝑠 = 𝑃𝑟𝑡 𝑡 = 4 years
𝐼𝑠 = (50000)(0.10)(0.75) (d) The unknown simple interest can be obtained by
𝐼𝑠 = 3,750 𝐼𝑠 = 𝑃𝑟𝑡
Answer. The simple interest charged is P 3,750. 𝐼𝑠 = (500,000)(0.125)(10)
𝐼𝑠 = 625,000
Example 4. Complete the table below to find the unknown.

2
ASIAN INSTITUTE OF COMPUTER STUDIES (AICS)

2.1.2 Maturity (Future Value) 𝐹 = 1,000,000 + 2,500


𝐹 = 1,002,500
Maturity (Future) Value
Method 2:
𝐹 = 𝑃 + 𝐼𝑠
𝐹 = 𝑃 (1 + 𝑟𝑡)
Where,
𝐹 = (1,000,000)(1 + 0.0025 (1))
𝐹 = maturity (future) value
𝐹 = 1,002,500
𝑃 = principal
Answer. The maturity or future value after 1 year is P1,002,500.
𝐼𝑠 = simple interest
(b) When 𝑡 = 5,
Substituting 𝐼𝑠 = 𝑃𝑟𝑡 gives:
Method 1:
𝐹 = 𝑃(1 + 𝑟𝑡)
𝐼𝑠 = 𝑃𝑟𝑡
Where,
𝐼𝑠 = (1,000,000)(0.0025)(5)
𝐹 = maturity (future) value
𝐼𝑠 = 12,500
𝑃 = principal
𝑟 = interest rate 𝐹 = 𝑃 + 𝐼𝑠
𝑡 = term/time in years 𝐹 = 1,000,000 + 12,500
𝐹 = 1,012,500
Example 5. Find the maturity value if 1 million pesos is deposited in a bank at an
annual simple interest rate of 0.25% after (a) 1 year (b) 5 years? Method 2:
Given. 𝐹 = 𝑃 (1 + 𝑟𝑡)
𝑃 = 1,000,000 𝐹 = (1,000,000)(1 + 0.0025 (5))
𝑟 = 0.25% = 0.0025 𝐹 = 1,012,500
Find: Answer. The maturity or future value after 5 years is P1,012,500.
(a) maturity or future value after 1 year
(b) maturity or future value after 5 years 2.1.3 Compound Interest
Solution.
(a) When 𝑡 = 1, the simple interest is given by: Maturity (Future) Value and Compound Interest
Method 1: 𝐹 = 𝑃(1 + 𝑟)𝑡
𝐼𝑠 = 𝑃𝑟𝑡 Where,
𝐼𝑠 = (1,000,000)(0.0025)(1) 𝑃 = principal or present value
𝐼𝑠 = 2,500 𝐹 = maturity (future) value at the end of the term
Method 2: 𝑟 = interest rate
The maturity or future value is given by:
𝑡 = term/time in years
𝐹 = 𝑃 + 𝐼𝑠

3
ASIAN INSTITUTE OF COMPUTER STUDIES (AICS)

(b) compound interest (𝐼𝑐 )


The Compound Interest 𝐼𝑐 is given by: Solution.
𝐼𝑐 = 𝐹 − 𝑃 (a) 𝐹 = 𝑃(1 + 𝑟)𝑡
Example 6. Find the maturity value and the compound interest if P20,000 is 𝐹 = (50,000)(1 + 0.5)0.5
compounded annually at an interest rate of 3% in 6 years? 𝐹 = 53,080.128
Given. (b) 𝐼𝑐 = 𝐹 − 𝑃
𝑃 = 20,000 𝐼𝑐 = 53,080.128 − 50,000
𝑟 = 3% = 0.03 𝐼𝑐 = 3,080.128
𝑡 = 6 years Answer. The future value is P53,080.13 and the compound interest is P3,080.13.
Find.
(a) maturity value (𝐹) Present Value at Compound Interest
(b) compound Interest (𝐼𝑐 ) 𝐹
𝑃= = 𝐹(1 + 𝑟)−𝑡
Solution. (1 + 𝑟)𝑡
(a) 𝐹 = 𝑃(1 + 𝑟)𝑡 Where,
𝐹 = (20,000)(1 + 0.03)6 𝑃 = principal or present value
𝑛−1
𝐹 = 23,881.0459 Studying the pattern, we𝐹 will
= maturity
derive (future) value at the
at the function end
𝑇𝑛 = of term
25(2) when 𝑛
𝑟 = is
represents the week, and this interest rate of exponential function.
an example
(b) 𝐼𝑐 = 𝐹 − 𝑃
𝑡 = term or time in years
𝐼𝑐 = 23,881.0459 − 20,000
𝐼𝑐 = 3,881.0459 Example 8. What is the present value of P30,000 due in 8 years if money is worth
Answer. The future value 𝐹 is P23,881.05 and the compound interest is P3,881.05 10% compounded annually?
(rounded off to the nearest hundredths). Given.
Example 7. An accountant for a corporation forgot to pay the firm’s income tax of 𝑃 = 30,000
𝑟 = 10% = 0.10
P50,000 on time. The government charged a penalty of 12.7% for the 6 months
𝑡 = 8 years
the money was late. Find the total amount that must be paid, and the compound
Find 𝑷.
interest charged.
Solution. The present value can be obtained by:
Given. 𝐹
𝑃 = 50,000 𝑃=
(1+𝑟)𝑡
𝑟 = 12.7% = 0.127 𝑃=
30,000
(1+0.10)8
𝑡 = 6 months = 0.5 year
𝑃 = 13,995.2214
Find.
Answer. The present value is P 13,995.22.
(a) maturity value (𝐹 )

4
ASIAN INSTITUTE OF COMPUTER STUDIES (AICS)

Example 9. A working student wants to deposit in a bank that pays 1.2% From lesson 2.1.3, the formula for the maturity value 𝐹 when principal 𝑃 is invested
compounded annually so that he will have P150,000 in 5 years. How much should at an annual interest rate 𝑗 compounded annually is 𝐹 = 𝑃(1 + 𝑗)𝑡 .
he deposit? 𝑖 (𝑚)
Because the rate of each conversion period is 𝑗 = , then in 𝑡 years, interest is
𝑚
Given.
compounded 𝑚𝑡 times. The following formula is obtained.
𝐹 = 150,000
𝑟 = 1.2% = 0.012 Maturity Value, Compounding 𝒎 times a year
𝑡 = 5 years 𝑚𝑡
𝑖 (𝑚)
Find 𝑷. 𝐹 = 𝑃 (1 + )
𝑚
Solution. The present value can be obtained by:
𝐹 Where,
𝑃=
(1+𝑟)𝑡 𝐹 = maturity (future) value
150,000
𝑃 = (1+0.012)5 𝑃 = principal
𝑃 = 136,347.494 𝑖 (𝑚) = nominal rate of interest (annual rate)
Answer. The student should deposit P 136,347.49. 𝑚 = frequency of conversion
𝑡 = term/time in years
2.1.4 Compounding More than Once a Year
Example 10. Find the maturity value and interest if P10,000 is deposited in a
bank at 2% compounded quarterly for 5 years.
Definition of Terms Given.
Frequency of conversion (𝒎) – a number of conversion periods in one year. 𝑃 = 10,000
Conversion or Interest Period – time between successive conversions of interest 𝑖 (4) = 0.02
Total Number of Conversion Periods (𝒏) – 𝑡 = 5 years
𝑛 = 𝑚𝑡 = (frequency of conversion) × (time in years) 𝑚 = 4 (quarterly)
Nominal Rate (𝒊(𝒎) ) – annual rate of interest Find.
Rate (𝒋) of interest for each conversion period (a) maturity (future) value (𝐹)
𝑖 (𝑚) 𝑎𝑛𝑛𝑢𝑎𝑙 𝑟𝑎𝑡𝑒 𝑜𝑓 𝑖𝑛𝑡𝑒𝑟𝑒𝑠𝑡 (b) compound interest (𝐼𝑐 )
𝑗= =
𝑚 𝑓𝑟𝑒𝑞𝑢𝑒𝑛𝑐𝑦 𝑜𝑓 𝑐𝑜𝑛𝑣𝑒𝑟𝑠𝑖𝑜𝑛
Solution.
(a) Compute for the interest rate in a conversion period by:
Note on Rate Notation. In earlier lessons, 𝑟 was used to denote the interest 𝑖 (4) 0.02
𝑗= = = 0.005
rate. Now that an interest rate can refer to two rates (either nominal or rate 𝑚 4
per conversion period), the symbol 𝑖 (𝑚) and 𝑗 will be used instead. Compute for the total number of conversion periods by:
𝑛 = 𝑚𝑡 = (4)(5) = 20 conversion periods
Compute for the maturity value using:

5
ASIAN INSTITUTE OF COMPUTER STUDIES (AICS)

𝐹 = 𝑃 (1 + 𝑗 )𝑛
𝐹 = (10,000)(1 + 0.005)20 Present Value 𝑷 at Compound Interest
𝐹 = 11,048.96 𝐹
𝑃=
(b) The compound interest is given by: 𝑖 (𝑚)
(1 + 𝑚 )𝑚𝑡
𝐼𝑐 = 𝐹 − 𝑃
Where,
𝐼𝑐 = 11,048.96 − 10,000
𝐹 = maturity (future) value
𝐼𝑐 = 1,048.96
𝑃 = principal
Example 11. Find the maturity value and interest if P10,000 is deposited in a bank 𝑖 (𝑚) = nominal rate of interest (annual rate)
at 2% compounded monthly for 5 years. 𝑚 = frequency of conversion
Given. 𝑡 = term/time in years
𝑃 = 10,000
𝑖 (12) = 0.02 Example 12. Find the present value of P50,000 due in 4 years if money is invested
𝑡=5 at 12% compounded semi-annually.
𝑚 = 12 (monthly) Given.
Find. 𝐹 = 50,000
(a) maturity (future) value (𝐹) 𝑡=4
(b) compound interest (𝐼𝑐 ) 𝑖 (2) = 0.12
Solution. Find 𝑷.
(a) Compute for the interest rate in a conversion period by: Solution.
𝑖 (12) 0.02 First, compute for the interest rate per conversion given by:
𝑗= = 𝑖 (2) 0.12
𝑚 12 𝑗= 𝑚
= 2
= 0.06
Compute for the total number of conversion periods by: The total number of conversion periods is:
𝑛 = 𝑚𝑡 = (12)(5) = 60 conversion periods 𝑛 = 𝑚𝑡 = (2)(4) = 8
Compute for the maturity value using: The present value can be computed by substituting these values in the
𝐹 = 𝑃(1 + 𝑗)𝑛 formula:
0.02
𝐹 = (10000)(1 + 12 )60 𝑃 = (1+𝑗)𝑛
𝐹

𝐹 = 11,050.79
Thus,
(b) The compound interest is given by: 50000 50000
𝑃 = (1+0.06)8 = (1+0.06)8 =P31,370.62
𝐼𝑐 = 𝐹 − 𝑃
𝐼𝑐 = 11,050.79 − 10,000
𝐼𝑐 = 1,050.79

6
ASIAN INSTITUTE OF COMPUTER STUDIES (AICS)

Example 13.
2.1.5 Simple Annuity
Consider a series of five $1,000 payments made at regular intervals.
Definition of Terms
Annuity – a sequence of payments made at equal (fixed) intervals or period of time.
Annuities can be classified as follows:

Annuities
According to payment Simple Annuity – an General Annuity – an
interval and interest annuity where the annuity where the
period payment interval is the payment interval is not
same as the interest the same as interest
period. period.
According to time of Ordinary Annuity (or Annuity Due – a type of
payment Annuity Immediate) – a annuity in which the
type of annuity in which payments are made at
the payments are made beginning of each
at the end of each payment interval.
Because of the time value of money—the concept that any given sum is worth
payment interval.
According to duration Annuity Certain – an Contingent Annuity – an more now than it will be in the future because it can be invested in the meantime—
annuity in which annuity in which the the first $1,000 payment is worth more than the second, and so on. So, let's assume
payments begin and end payments extend over that you invest $1,000 every year for the next five years, at 5% interest. Below is
at definite times. an indefinite (or how much you would have at the end of the five-year period.
indeterminate) length of
time.

Term of an Annuity (𝒕) – time between the first payment interval and last payment
interval

Regular or Periodic Payment (𝑹) – the amount of each payment

Amount (Future Value) of an Annuity (𝑭) – sum of future values of all the
payments to be made during the entire term of the annuity.

Present Value of an Annuity (𝑷) – sum of present values of all the payments to be
made during the entire term of the annuity.

7
ASIAN INSTITUTE OF COMPUTER STUDIES (AICS)

Rather than calculating each payment individually and then adding them all up,
however, you can use the following formula, which will tell you how much money
you'd have in the end:

Amount (Future Value) of an Ordinary Annuity


The future value 𝐹 of an ordinary annuity
(1 + 𝑗)𝑛 − 1
𝐹=𝑅
𝑗
Where,
𝑅 = regular payment
𝑗 = interest rate per period
𝑛 = number of payments

(1+𝑗)𝑛−1
𝐹=𝑅 𝑗
(1+0.05)5 −1
𝐹 = (1000) ( )
0.05
𝐹 = 5,525.63 We have this formula in getting the present value of an ordinary annuity:
Answer. At the end of the five-year period, you will have $5,525.63.

In contrast to the future value calculation, a present value 𝑃 calculation tells you Present Value of an Ordinary Annuity
how much money would be required now to produce a series of payments in the The present value of an ordinary annuity is given by:
future, again assuming a set interest rate. 1 − (1 + 𝑗)−𝑛
𝑃=𝑅
𝑗
Using the same example of five $1,000 payments made over a period of five years, Where,
here is how a present value calculation would look. It shows that $4,329.58, 𝑅 = regular payment
invested at 5% interest, would be sufficient to produce those five $1,000 payments. 𝑗 = interest rate per period
By substituting it to the formula we’ll have:
𝑛 = number of payments

1 − (1 + 𝑗)−𝑛
𝑃=𝑅
𝑗
1−(1+0.05) −5
𝑃 = (1,000)( )
0.05

8
ASIAN INSTITUTE OF COMPUTER STUDIES (AICS)

𝑃 = 4,329.48
2.1.6 General Annuity
Answer. P4,239.48
Recall:
The cash value or cash price of a purchase is equal to the down payment (if General Annuity – an annuity where the length of the payment interval is not
there is any) plus the present value of the installment payments.
the same as the length of interest compounding period.

Example 14. Mr. Lozada paid P200,000 as down payment for a car. The remaining General Ordinary Annuity – a general annuity in which the periodic payment is
amount is to be settled by paying P16,200 at the end of each month for 5 years. If made at the end of the payment interval.
interest is 10.5% compounded monthly, what is the cash price of his car?
Given. Examples of General Annuity:
Down Payment = 200,000 1. Monthly installment payment of car, lot, or house with an interest rate that
𝑅 = 16,200 is compounded annually.
𝑖 (12) = 0.105 2. Paying a debt semi-annually when the interest is compounded monthly.
𝑚 = 12
0.105 Future and Present Value of a General Ordinary Annuity
𝑗= 12
= 0.00875 The future value 𝐹 and present value 𝑃 of a general annuity is given by:
𝑡 = 5 years (1+𝑗) 𝑛−1 1−(1+𝑗)−𝑛
𝐹=𝑅 𝑃=𝑅
𝑗 𝑗
𝑛 = 𝑚𝑡 = (12)(5) = 60 periods
Find Cash Value or Cash Price of the car. Where,
Solution. 𝑅 = regular payment
The present value of this ordinary annuity is given by: 𝑗 = equivalent interest rate per payment interval
1−(1+𝑗)−𝑛 converted from the interest rate per period;
𝑃=𝑅 𝑗 𝑛 = number of payments
1−(1+0.00875)−60
𝑃 = (16,200)( )
0.00875 The formulas for 𝐹 and 𝑃 are the same as those in Lesson 2.1.5. The extra step
𝑃 = 753,702.20
occurs in finding 𝑗: the given interest rate per period must be converted to an
𝐶𝑎𝑠ℎ 𝑉𝑎𝑙𝑢𝑒 = 𝐷𝑜𝑤𝑛 𝑃𝑎𝑦𝑚𝑒𝑛𝑡 + 𝑃𝑟𝑒𝑠𝑒𝑛𝑡 𝑉𝑎𝑙𝑢𝑒 equivalent rate per payment interval.
= 200,000 + 753,702.20
Examples 15. Robert started to deposit P1,000 monthly in a fund that pays 6%
= 953,702.20
compounded quarterly. How much will be in the fund after 15 years?
Answer. The cash price of the car is P953,702.20
Given.
𝑅 = 1,000
𝑛 = (12)(15) = 180 payments

9
ASIAN INSTITUTE OF COMPUTER STUDIES (AICS)

𝑖 (4) = 0.06𝑚 = 4
Number of Decimal Places
Find 𝑭.
When solving for an equivalent rate, say 𝑗 = (1.015)1⁄3 − 1 in example 15, six
Solution.
or more decimal places will be used. If you use fewer or more decimal places,
1. Convert 6% compounded quarterly to its equivalent interest rate for
your answers may differ from the answers provided in the text. You can ignore
monthly payment interval.
these discrepancies, but it is suggested that you use at least six decimal places,
𝐹1 = 𝐹2
or the exact value.
𝑖 (12) (12)𝑡 𝑖4
𝑃(1 + ) = 𝑃(1 + )(4)𝑡
12 4 Example 16. Bob borrowed an amount of money from Joy. He agrees to pay the
𝑖 (12) 12 0.06 4
(1 + 12 ) = (1 + 4 ) principal plus interest by paying P38,973.76 each year for 3 years. How much
12
𝑖 (12) money did he borrow if interest is 8% compounded quarterly?
(1 + ) = (1.015)4
12 Given.
𝑖 (12) 𝑅 = 38,973.76
1+ 12
= [(1.015)4 ](1⁄12 )
𝑖 (12)
𝑖 (4) = 0.08
= (1.015)1⁄3 − 1 𝑚=4
12
𝑖 (12) 𝑛 = 3 payments
12
= 0.00497521 = 𝑗
Find Present Value 𝑷.
Thus, the interest rate per monthly payment interval is 0.00497521 or Solution.
0.497521%. 1. Convert 8% compounded quarterly to it equivalent interest rate for each
payment interval.
2. Apply the formula in finding the future value of an ordinary annuity using 𝐹1 = 𝐹2
the computed equivalent rate. 𝑖 (1) (1)𝑡 𝑖 (4) 4𝑡
(1 + 𝑗 )𝑛 − 1 𝑃(1 + ) = 𝑃(1 + )
𝐹=𝑅 1 4
𝑗 𝑖 (1) 0.08 4
(1 + 0.00497521)180 − 1 (1 + ) = (1 + )
𝐹 = (1000)( 1 4
0.00497521
𝑖 (1)
(1 + ) = (1 + 0.02)4
𝐹 = 290,082.51 1
Thus, Robert will have P290,082.51 in the fund after 20 years. 𝑖 (1)
= (1.02)4 − 1
1
𝑖 (1)
= 𝑗 = 0.082432 = 8.24%
1

Thus, the interest rate per payment interval is 0.082432 or 8.24%.

10
ASIAN INSTITUTE OF COMPUTER STUDIES (AICS)

2. Apply the formula in finding the present value of an ordinary annuity using Example 17. Mr. Jose received two offers on a lot that he wants to sell. Mr. Dellosa
the computed equivalent rate 𝑗 = 0.082432. has offered P50,000 and P1 million lump sum payment 5 years from now. Mr.
1 − (1 + 𝑗)−𝑛 Monsalud has offered P50,000 plus P40,000 every quarter for five years. Compare
𝑃=𝑅
𝑗 the fair market values of the two offers if money can earn 5% compounded
1 − (1 + 0.082432)−3 annually. Which offer has a higher market value?
𝑃 = 38973.76
0.082432 Given.
𝑃 = 100,000.0 Mr. Dellosa’s offer: Mr. Monsalud’s offer:
Hence, Bob borrowed P100,000 from Joy. P50,000 down payment P50,000 down payment
P1,000,000 after 5 years P40,000 every quarter for 5 years.

2.1.7 Cash Flow and Fair Market Value Find Fair Market Value (FMV) of each offer.
Solution.
A cash flow is a term that refers to payments received (cash inflows) or We illustrate the cash flows of the offers using time diagrams
payments or deposit made (cash outflows). Cash inflows can be represented by
positive numbers and cash outflows can be represented by negative numbers. Mr. Dellosa’s offer:

50,000 1 million
The fair market value or economic value of a cash flow (payment stream) on a
particular date refers to a single amount that is equivalent to the value of the 0 1 2 3 4 5
payment stream at that date. This particular date is called the focal date.
Mr. Monsalud’s offer:

50,000
40,000 40,000 40,000 … 40,000

0 1 2 3 … 20

Choose the focal date and determine the values of the two offers at the focal date.
For example, the focal date can be the date at the start of the term.

Since the focal date is at 𝑡 = 0, compute for the present value of each offer.

Mr. Dellosa’s offer: Since P50,000 is offered today, then its present value is still
Php50,000. The present value of P1,000,000 offered 5 years from now is:

11
ASIAN INSTITUTE OF COMPUTER STUDIES (AICS)

𝑃 = 𝐹(1 + 𝑗)−𝑛 𝐹𝑀𝑉 = P755,572.70


Hence, Mr. Dellosa’s offer has a higher market value. The difference between the
𝑃 = 1,000,000(1 + 0.05)−5
market values of the two offers at the start of the term is
𝑃 = P783,526.20 833,526.20 − 755,572.70 = P77,953.50.

𝐹𝑎𝑖𝑟 𝑀𝑎𝑟𝑘𝑒𝑡 𝑉𝑎𝑙𝑢𝑒 (𝐹𝑀𝑉) = 𝑑𝑜𝑤𝑛 𝑝𝑎𝑦𝑚𝑒𝑛𝑡 + 𝑝𝑟𝑒𝑠𝑒𝑛𝑡 𝑣𝑎𝑙𝑢𝑒 Alternate Solution (Focal date at the end of the term)
Mr. Dellosa’s offer:
= 50,000 + 783,526.20
The future value of P1,000,000 at the end of term at 5% compounded
𝐹𝑀𝑉 = P833,526.20 annually is given by:
𝐹 = 𝑃(1 + 𝑗)𝑛
Mr. Monsalud’s offer: 𝐹 = 50,000(1 + 0.05)5
We first compute for the present value of a general annuity with quarterly 𝐹 = 63,814.08
payments but with annual compounding at 5%. The fair market value of this offer at the end of the term is 63,814.08 plus
1 million pesos amounting to P1,063,814.08.
Solve the equivalent rate, compounded quarterly, of 5% compounded annually.
Mr. Monsalud’s offer:
𝐹1 = 𝐹2 The future value of this ordinary general annuity is given by:
(1+𝑗)𝑛−1
𝑖 (4) (4)(5) 𝑖 (1) 𝐹=𝑅
𝑃(1 + ) = 𝑃(1 + )(1)(5) 𝑗
4 1
(1+0.012272)20 −1
20
𝐹 = 4,000
0.012272
𝑖 (4) 0.05 5
(1 + ) = (1 + ) 𝐹 = 900,509.40
4 1
The future of P50,000 at the end of the term is P63,814.08, which was
𝑖 (4) already determined earlier.
1+ = (1.05)(1⁄4)
4
𝑖 (4)
𝐹𝑎𝑖𝑟 𝑀𝑎𝑟𝑘𝑒𝑡 𝑉𝑎𝑙𝑢𝑒 = 900,509 + 63,814.08 = P964,323.48.
= (1.05)(1⁄4) − 1 As expected, Mr. Dellosa’s offer still has a higher market value, even if the focal
4
𝑖 (4) date was chosen to be at the end of the term. The difference between the market
4
= 0.012272
values of the two offers at the end of the term is:
The present value of an annuity is given by:
1−(1+𝑗) −𝑛
𝑃=𝑅 1,063,814.08 − 964,323.48 = P964,323.48
𝑗
1−(1+0.012272)−20
You can also check that the present value of the difference is the same as the
𝑃 = 4000 difference computed when the focal date was the start of the term:
0.012272
𝑃 = P705,572.70 P99,490.60(1 + 0.05)−5 = P77,953.49
𝐹𝑎𝑖𝑟 𝑀𝑎𝑟𝑘𝑒𝑡 𝑉𝑎𝑙𝑢𝑒 (𝐹𝑀𝑉) = 𝑑𝑜𝑤𝑛 𝑝𝑎𝑦𝑚𝑒𝑛𝑡 + 𝑝𝑟𝑒𝑠𝑒𝑛𝑡 𝑣𝑎𝑙𝑢𝑒
= 50,000 + 705,572.70

12
ASIAN INSTITUTE OF COMPUTER STUDIES (AICS)

2.1.8 Deferred Annuity Example 18. A credit card company offers a deferred payment option for the
purchase of any appliance. Roxanne plans to buy a smart television set with
Definition of Terms monthly payments of P4,000 for 2 years. The payments will start at the end of 3
months. How much is the cash price of the TV set if the interest rate is 10%
Deferred Annuity – an annuity that does not begin until a given time interval has
passed. compounded monthly?
Given.
Period of Deferral – time between the purchase of an annuity and the start of the 𝑅 = 4,000
payments for the deferred annuity. 𝑚 = 12
𝑖 (12) = 10
Time Diagram for a Deferred Annuity Find 𝑷.
R* R*… R* R R… R Solution. The annuity is deferred for 2 months and it will go on for 2 years. The first
payment is due at the end of 3 months, or at the end 3rd conversion period.
0 1 2… k k+1 k+2 k+n Thus, there are 2 artificial payments.
Number of artificial payments: 𝑘 = 2
In this time diagram the period of deferral is 𝑘 because the regular payments of Number of actual payments: 𝑛 = 𝑚𝑡 = (12)(2) = 24
𝑅 start at the time 𝑘 + 1. 𝑖 (12) 0.10
𝑗= = = 0.00833
The notation 𝑅∗ represent 𝑘 “artificial payments”, each equal to 𝑅, but are not 𝑚 12
Interest rate per period:
actually paid during the period of deferral.
If you assume that there are payments in the period of deferral, there would be a
total of 𝑘 + 𝑛 = 2 + 24 = 26 payments.
To determine the present value of a deferred annuity, find the present value of all Time Diagram:
𝑘 + 𝑛 payments (including the artificial payments), then subtract the present value 4000 4000 … 4000
of all artificial payments.
0 1 2 3 4 … 26
Present Value of a Deferred Annuity
The present value of a deferred annuity is given by:
Thus, the present value of the deferred annuity can be solved as:
1 − (1 + 𝑗)−(𝑘+𝑛) 1 − (1 + 𝑗)−𝑘
𝑃=𝑅 −𝑅
𝑗 𝑗 1−(1+𝑗)−(𝑘+𝑛) 1−(1+𝑗)−𝑘
𝑃=𝑅 𝑗
−𝑅 𝑗
Where:
1−(1+0.00833)−26 1−(1+0.00833) −2
𝑅 = regular payment = 4000 − 4000 = 85,260.53
0.00833 0.00833
𝑗 = interest rate per period
𝑛 = number of payments Therefore, the present value of these monthly pensions is P85,260.53.
𝑘 = number of conversion periods in the deferral

13
ASIAN INSTITUTE OF COMPUTER STUDIES (AICS)

REFERENCES
Books
Department of Education, (2016) General Mathematics Learner’s Material
First Edition 2016, Lesson 23-30, pp. 139-212

Vibal Inc., Chan, J.H.N., Miro, E.D., Quiming, R.S., (2016) General
Mathematics; Chapter 2 - Lesson 2.1-2.3 Interest and Future Value, Present
Value, Annuities, pp. 66-85

Electronic Sources

Heyford, S.C., Calculating Present and Future Value of Annuities, updated


Apr. 30, 2020retrieved from
https://www.investopedia.com/retirement/calculating-present-and-
future-value-of-annuities/

Kagan, J., Deferred Annuity, updated Jan. 13, 2020, retrieved from
https://www.investopedia.com/terms/d/deferredannuity.asp

Pinegar, K., Exercises – Simple and Compound Interest, Module 2:


Mathematics of Finance retrieved from
https://courses.lumenlearning.com/sanjacinto-finitemath1/chapter/4-2-
exercises-simple-and-compound-interest/

Understanding Cash Flow: Cash Inflow and Outflow Image


https://images.app.goo.gl/EzTFVhtMNYSj7DGR9

14
ASIAN INSTITUTE OF COMPUTER STUDIES (AICS)

a simple interest basis. Bank B, meanwhile, offers an interest rate of 7%


Activity 1: INTERESTing! per year for 3 years. Which bank’s loan product should he choose?

Directions. Solve the following problems involving simple and compound interests.
6. If you borrow P3,000 at 14% simple interest for 10 months, how much will
State the given per problem and show your complete solution always.
you owe in 10 months? How much interest will you pay?
1. When invested at an annual interest rate of 7%, the amount earned
P11,200 of simple interest in two years. How much money was originally
invested?
7. Grandparents deposited P6,000into a grandchild’s account toward a
college education. How much money (to the nearest peso) will be in the
account 17 years from now if the account earns 9% compounded
2. If an entrepreneur applies for a loan amounting to P500,000 in a bank, the
monthly?
simple interest of which is P157,500 for 3 years, what interest rate is being
charged?

8. If an investor wants to earn an annual interest rate of 10.76% on a 26-


3. How long will a principal earn an interest equal to half of it at 5% simple
week bill with a maturity value of P5,000, how much should the investor
interest?
pay for the bill?

4. After receiving her retirement pay of P10,000,000, Hazel is approached by


two banks offering slightly different deposit products. Bank A is offering an
9. Robert received P7280 as a simple interest rate after investing P52,000 in
interest rate of 5% with the interest computed on a simple basis. On the
other hand, Bank B offers a lower rate of 4.5%, compounded annually. If a bank 2 years ago. How much interest rate does the bank offers?
Hazel intends to keep the deposit for the next 10 years, which bank offers
the better product?

10. Jam received P10,500 as a simple interest from the P40,000 she invested
5. Mark wants to buy a car worth P800,000. He currently has P300,000 and in a bank that offers 3.5% annual interest rate. How many years did Jam
invested?
plans to borrow the remaining P500,000 through the bank. Bank A offers
an interest rate of 8% per year for 3 years where interest is computed on

15
ASIAN INSTITUTE OF COMPUTER STUDIES (AICS)

7. Person (or institution) who owes the money or avails of the funds from the
Activity 2: Scavenger Hunt lender.
8. Amount after t years that the lender receives from the borrower on the
maturity date.
Directions. Match the icons below (with assigned names) with their definition. 9. Interest that is computed on the principal and then added to it.
Refer to the previous lessons to look for the answer. 10. Annual rate, usually in percent, charged by the lender, or rate of increase
of the investment.

Activity 3: Being Thrifty

Simple Debtor Principal Annuity Term


Interest Directions. Solve the following problems involving ordinary annuity and cash value.
State the given per problem and show your complete solution always.

1. In order to save for her high school graduation, Alison decided to save
P200 at the end of each month. If the bank pays 0.250% compounded
monthly, how much will be her money at the end of 6 years?

Maturity Origin (Loan) Lender Rate Cash Flow


(Future) Date
Value 2. Aling Katring started to deposit P2,000 quarterly in a fund that pays 5.5%
compounded quarterly. How much will be in the fund after 6 years?

1. A term that refers to payments received.


2. Person (or institution) who invests the money or makes the fund available.
3. Amount of time in years the money is borrowed or invested; length of time 3. The buyer of a house and lot pays P200,000 cash and P10,000 every
between the origin and maturity dates. month for 20 years. If money is 9% compounded monthly, how much is
4. A sequence of payments made at equal (fixed) intervals or period of time. the cash value of the lot?
5. Date on which the money is received by the borrower.
6. Amount of money borrowed or invested on the origin date

16
ASIAN INSTITUTE OF COMPUTER STUDIES (AICS)

CHAPTER 2.2 STOCKS, BONDS, AND LOANS 


KEY QUESTIONS
2ND QUARTER

What is the difference


MODULE 10
WEEK _____

At the end of this module, you are expected to: between stocks and bonds?
 Illustrate and distinguish stocks and bonds,  How do you read stock
 Analyzes the different market indices for stocks and bonds, tables?
 Differentiate business and consumer loans,  What are the types of loans?
 Solves problems involving business and consumer loans (amortization, mortgage).

2.2.1 Stocks and Bonds bondholders are lenders to the institution which may be a government or private
Stocks are shares in the ownership of the company. Owners of stocks may be company. Some bond issuers are the national government, government agencies,
considered as part-owners of the company. government owned and controlled corporations, non-bank corporations, banks
and multilateral agencies. Bondholders do not vote in the institution’s earnings. On
Two Types of Stocks the maturity date, the bondholders will receive the face amount of the bond.
Common Stock Preferred Stock
Common stock presents shares of Preferred stock comes with no voting
ownership in a corporation and the rights.
type of stock in which most people Preferred stock functions similarly to
invest. bonds since with preferred shares,
Common shares represent a claim on investors are usually guaranteed a
profits (dividends) and confer voting fixed dividend in perpetuity.
rights.

Both will receive dividends or share of earnings of the company. Dividends are paid
first to preferred shareholders.

Stocks can be bought or sold at its current price called the market value. When a
person buys some shares, the person receives a certificate with the corporation’s
name, owner’s name, number of shares and par value per share.

Bonds are interest bearing security which promises to pay amount of money on a
certain maturity date as stated in the bond certificate. Unlike stockholders,

17
ASIAN INSTITUTE OF COMPUTER STUDIES (AICS)

Definition of Terms in Relation to Stocks Dividend Percentage = 4%


Stocks – share in the ownership of a company. Par Value = P450
Number of Shares = 250
Dividend – share in the company’s profit.
Find Dividend.
Dividend Per Share – ratio of the dividends to the number of shares. Solution.
The dividend per share is: P450 × 0.04 =P18.
Stock Market – a place where stocks can be bought or sold. The stock market in Since there are 250 shares, the total dividend is:
the Philippines is governed by the Philippine Stock Exchange (PSE). P18/share × 250 shares = P4,500
Market Value – the current price of the stock at which it can be sold. In summary,
Dividend = (Dividend Percentage)×(Par Value)×(No. of Shares)
Stock Yield Ratio – ratio of the annual dividend per share and the market value per = (0.04)(450)(250)
share. Also called current stock yield. = 4,500
Thus, the dividend is P4,500.
Par value – the per share amount as stated on the company certificate. Unlike
market value, it is determined by the company and remains stable over time. Example 3. Corporation ABC, with a current market value of P67, gave a dividend
Example 1. AMD Bank declared a P50,000,000 dividend for the common stocks. If of P7 per share for its common stock. Corporation EFG, with a current value of P85,
gave a dividend of P12 per share. Use the stock yield ratio to measure how much
there are total of 600,000 shares of a common stock, how much is the dividend per
share? dividends shareholders are getting in relation to the amount invested.
Solution.
Given.
Given. Corporation ABC:
Total Dividend = P50,000,000
Total Shares = P600,000 Dividend per Share = P7
Market Value = P67
Find Dividend per Share.
Find Stock Yield Ratio.
Solution. 𝐷𝑖𝑣𝑖𝑑𝑒𝑛𝑑 𝑝𝑒𝑟 𝑆ℎ𝑎𝑟𝑒
𝑇𝑜𝑡𝑎𝑙 𝐷𝑖𝑣𝑖𝑑𝑒𝑛𝑑 =
Dividend per Share = 𝑇𝑜𝑡𝑎𝑙 𝑆ℎ𝑎𝑟𝑒𝑠
𝑀𝑎𝑟𝑘𝑒𝑡 𝑉𝑎𝑙𝑢𝑒
7
50,000,000 =
= 67
600,000
= 83.33 = 0.1045 = 10.45%
Therefore, the dividend per share is P83.33.
Corporation EFG:
Example 2. Mrs. Dizon owns 250 shares of stock with a par value of P450 in KMC Dividend per Share = P12
Corporation. If the corporation declared a 4% dividend on a stock with a par value Market Value = P85
of P450, How much is the dividend she received? Find Stock Yield Ratio.
Given. 𝐷𝑖𝑣𝑖𝑑𝑒𝑛𝑑 𝑝𝑒𝑟 𝑆ℎ𝑎𝑟𝑒
= 𝑀𝑎𝑟𝑘𝑒𝑡 𝑉𝑎𝑙𝑢𝑒

18
ASIAN INSTITUTE OF COMPUTER STUDIES (AICS)

12
= 85
If 𝑃 = 𝐹, the bond is purchased at par.
= 0.1412 = 14.12% If 𝑃 < 𝐹, the bond is purchased at a discount.
Corporation EFG has a higher stock-yield-ratio than Corporation ABC. Thus, each If 𝑃 > 𝐹, the bond is purchased at premium.
peso will earn you more if you invest in Corporation EFG than in Corporation ABC. Term of a Bond – fixed period of time (in years) at which the bond is redeemable
If all other things are equal, then it is wiser to invest in Corporation EFG. as stated in the bond certificate; number of years from time of purchase to
maturity date.
As Example 3 shows, stock yield ratio can be used to compare two or more
investments. Fair Price of a Bond – present value of all cash inflows to the bondholder.

Example 4. Calculate the amount of the semi-annual coupon for a bond with a face
Definition of Terms in Relation to Bonds value of P450,000 that pays 9%, payable semi-annually for its coupons.
Bond – interest-bearing security which promises to pay (1) a stated amount of Given.
money on the maturity date, and (2) regular interest payments called coupons. Face Value 𝐹 = 450,000
KEY TAKEAWAYS Coupon Rate 𝑟 = 9%
 Bonds are units of corporate debt issued by companies and securitized Find Amount of the Semi-annual Coupon.
as tradeable assets. Solution.
 A bond is referred to as a fixed income instrument since bonds Annual coupon amount: 450,000(0.09) = 40,500
1
traditionally paid a fixed interest rate (coupon) to debtholders. = 40,500 ( ) = 20,250
2
Variable or floating interest rates are also now quite common. Thus, the amount of the semi-annual coupon is P20,250.
 Bond prices are inversely correlated with interest rates: when rates go
up, bond prices fall and vice-versa. Note: The coupon rate is used only for computing the coupon amount, usually
 Bonds have maturity dates at which point the principal amount must paid semi-annually. It is not the rate at which money grows. Instead current
be paid back in full or risk default. market conditions are reflected by the market rate, and is used to compute the
 present value of future payments.
Coupon – periodic interest payment that the bondholder receives during the time
between purchase date and maturity date; usually received semi-annually. Example 5. A bond has a face value of P100,000 and its maturity date is 10 years
from now. The coupon rate is 5% payable semi-annually. Find the fair price of this
Coupon Rate – the rate per coupon payment period; denoted by 𝑟. bond, assuming that the annual market rate is 4%.
Price of a Bond – the price of the bond at purchase time; denoted by 𝑃. Given.
Coupon rate 𝑟 = 5%, payable semi-annually
Par Value or Face Value – the amount payable on the maturity date, denoted by Face Value = 100,000
𝐹. Time to Maturity = 10 years

19
ASIAN INSTITUTE OF COMPUTER STUDIES (AICS)

Number of periods = 2(10) = 20 method (e.g., simple average, weighted average, etc.). This is then tracked as an
Market rate = 4% attempt to describe the health of the chosen portfolio via a single index value.
Solution.
For instance, the Philippine Stock Exchange Index (PSEi) was formed to represent
Amount of semi-annual coupon:
0.05 the overall health of the Philippine Stock Market. When the PSEi increases, it does
100,000 ( 2
) = 2,500 not necessarily mean that all the stocks in the index went up, but indicates that
The bondholder receives 20 payments of P2,500 each, and P100,000 at 𝑡 = 10. either (1) most of stocks did increase, or (2) some of the stocks significantly went
Present Value of P100,000: up.
𝐹 100,000
𝑃 = (1+𝑗)𝑛 = (1+0.04)10 = 67,556.42
Stock Index Tables
Present Value of 20 payments at P2,500 each: Stock indices are reported in the business section of magazines or newspapers, as
Convert 4% to equivalent semi-annual rate: well as online (https://www.pse.com.ph/stockMarket/home.html). The following
2
𝑖 (2) table shows how a list of index values is typically presented:
(1 + 0.04)1 = (1 + )
2
𝑖 (2)
2
= 0.019804
1−(1+𝑗)−𝑛 1−(1+0.019804)−20
𝑃=𝑅 = 2500 = 40,956.01
𝑗 0.019804
Price = 67,556.42 + 40,956.01 = 108,512.43
Thus, a price of P108,512.14 is equivalent to all future payments, assuming an
annual market rate of 4%.

2.2.2 Market Indices for Stocks and Bonds


A stock market index is a measure of a portion of the stock market.

With the numerous stocks listed in the market, it is quite impossible to assess the
market’s health by looking into each stock. In response to this need, indices were
created to draw up to track a close representation of market performance. By
definition, an index measure change. Just like the Consumer Price Index which Value – value of the index
tracks the performance o a hypothetical collection of securities, most common of
which are bonds and stocks. Chg – change of the index value from the previous trading day (i.e., value today
minus value yesterday)
Basically, a portfolio of securities is first form based on a pre-determined selection
criteria. An index value is then computed by using a pre-determined calculation %Chg – ratio of Chg to Val (i.e., Chg divided by Value)

20
ASIAN INSTITUTE OF COMPUTER STUDIES (AICS)

Stock Tables Bid Ask/Offer


Various information about stock prices can be reported. The following table shows Size Price Price Size
how information about stocks can be represented (values are hypothetical). 122 354,100 21.6000 21.8000 20,000 1
52- 52- STOCK HI LO DIV VOL CLOSE NETCHG 9 81,700 21.5500 21.9000 183,500 4
WK HI WK (100s) 42 465,500 21.5000 22.1500 5,100 1
LOW 2 12,500 21.4500 22.2500 11,800 4
25.00 21.00 AAA 23.00 22.00 1.00 1894 22.00 0.10 9 14,200 21.4000 22.3000 23,400 6
83.00 33.00 JJJ 79.00 65.00 2.35 7601 33.50 -0.2
In the table, the terms mean the following:
52-Week High – this column gives you the highest price that particular stock has Bid Size – the number of individual buy orders and the total number of shares they
reached in the most recent 52-week period. wish to buy.
Bid Price – the price these buyers are willing to pay for the stock.
52-Week Low – this column gives you the lowest price that the particular stock has Ask Price – the price the sellers of the stock are willing to sell the stock.
reached in the most recent 52-week period. Ask Size – how many individual sell orders have been placed in the online platform
HI/LO – highest/lowest selling price of the stock in the last trading day. and the total number of shares these sellers wish to sell.

STOCK – three-letter symbol the company is using for trading. Bond Market Indices
A bond market index is a measure of a portion of the bond market.
DIV – dividend per share last year.
Bonds market (or debt market) is where all bonds are issued for the first time and
VOL (100s) – number of shares (in hundreds) traded in the last trading day. In this traded subsequently. The main platform for bonds or fixed income securities in the
case, stock AAA sold 1894 shares of 100 which is equal to 18,940 shares. Philippines is the Philippine Dealing and Exchange Corporation (PDEx). Unlike stock
CLOSE – closing price on the last trading day. indices which are associated with virtually every stock market in the world, bond
market indices are far less common. In fact, other than certain regional bond
NETCHG – net change between the last two trading days. In the case of AAA, the indices which have sub-indices covering the Philippines, our bond market does not
net change is 0.10. The closing price the day before the last trading day is typically compute a bond market index. Instead, the market rates produced from
P22.00 −P0.10 = P12.00. the bond market are interest rates which may be used as benchmarks for other
financial instruments.
Buying or Selling Stocks
To buy or sell stocks, one may go to the PSE personally. However, most transactions The Bond Market and the Government Bonds
nowadays are done by making a phone call to a registered broker by logging on to A primary market is the avenue where a financial product is first sold into the
a reputable online trading platform. Those with accounts in online trading market. Being the first change-of-hand, all proceeds from the sale will go to the
platforms may often encounter a table such as the following. originator of the product. Hence, this is where governments and corporations raise
funds either via debt or equity.

21
ASIAN INSTITUTE OF COMPUTER STUDIES (AICS)

Government bonds are auctioned out to banks and other brokers and dealers every For stocks RRR and LLL:
Monday by the Bureau of Treasury. Depending on their terms (or tenors), these 1. What was the lowest price of the stock for the last 52 weeks?
bonds are also called treasury bills (t-bills), treasury notes (t-notes), or treasury 2. What was the dividend per share last year?
bonds (t-bonds). The resulting coupon rates and the total amount sold for these 3. What was the annual percentage yield last year?
bonds are usually reported by news agencies on the day right after the auction. 4. What was the closing price in the last trading day?
Since these bond transactions involve large amounts, these bonds are usually 5. What was the closing price the day before the last trading day?
limited to banks, insurance films, and other financial institutions. The banks may
Answers.
then re-sell these bonds to its clients.
For stock RRR:
Although the coupon rate for bonds is fixed, bond prices fluctuate because they 1. Lowest price = P105.00
are traded among investors in what is called the secondary market. This is the 2. Dividend per Share = P3.50
avenue where all financial products are re-sold in the market amongst investors. 3. YLD% = 2.8%
Consequently, all proceeds from the sale goes to the seller of the product and not 4. Closing price = P118.50
the original issuer. For instance, proceeds from the sale of a common stock of a 5. Closing price (the day before the last trading day) =
firm in the secondary market will go to the holder of the stock and not the firm P118.50 +P0.50 =P119.00
itself.
For stock LLL:
The main purpose of the secondary market is to be able to give liquidity to the 1. Lowest price = P12.00
investors. Consequently, firms also use the secondary market as a barometer on 2. Dividend per Share = P0.90
their performance and also a yardstick to how much their firm is worth at the 3. YLD% = 1.1%
moment. 4. Closing price = P15.80
5. Closing price (the day before the last trading day) =P15.80 −P0.10 =
Despite the fact the bond investing is considered safer than stock investing, there
P15.70
is still some risk involved. The most extreme scenario is default by the issuer. In this
case, the investor can lose not only the coupons, but even the money invested in
2.2.3 Business and Consumer Loans
the bond. Bond investors should thus be aware of the financial condition of the
issuer of the bond and of the prevailing market conditions.
Definition of Terms
Example 6. Consider the following listing on stocks and answer the question that Business Loan – money lent specifically for a business purpose. It may be used to
follow: start a business or to have a business expansion.

52-WK 52-WK STOCK DIV YLD% VOL CLOSE NETCHG Consumer Loan – money lent to an individual for a personal or family purpose.
HI LOW (100s)
120 105 RRR 3.5 2.8 4050 118.50 -0.50 Collateral – assets used to secure the loan. It may be real-estate or other
16 12 LLL 0.9 1.1 1070 15.80 0.10 investments.

22
ASIAN INSTITUTE OF COMPUTER STUDIES (AICS)

Term of the loan – time to pay the entire loan. Example 12. For the expansion of her business, Ms. Viaña borrowed P500,000. The
effective rate of interest is 8%. The loan is to be repaid in full after one year. How
In example 7-11, identify whether the following is a consumer or a business loan. much is to be paid after one year?
Example 7. Mr. and Mrs. Lozada wants to borrow money from the bank to finance Given.
𝑃 = 500,000
the college education of their son, Asher. Answer: Consumer loan
𝑗 = 0.08
Example 8. As a newly graduate, Robert plans to have a burger stall. He wants to 𝑛=1
borrow some money from the bank in order for him to buy the equipment and Find 𝑭.
furniture for the burger stall. Answer: Business loan Solution.
𝐹 = 𝑃(1 + 𝑗)𝑛 = 500,000(1 + 0.08)1 = 540,000
Example 9. Mark will graduate this coming August. His parents are planning to Thus, an amount of P540,000 must be paid after one year.
apply for a loan to purchase a car for him as a graduation gift. Answer: Consumer
Example 13 (Chattel Mortgage). Ryan borrowed P1,500,000 for the purchase of a
loan car. If his monthly payment is P35,000 on a 5-year mortgage, find the total amount
Example 10. Anneth runs a courier business. She wants to have an additional of 9 of interest.
Given.
motorcycles for the expansion of his business. She applied for a loan in a bank.
P = 1,500,000
Answer: Business loan
Monthly payment = 35,000
Example 11. The soon-to-be Mr. and Mrs. Deraja applied for a loan to finance their Solution.
wedding expenses. Answer: Consumer loan The total amount paid is given by:
Total amount = (35,000)(12 months)(5years)
2.2.4 Amortization and Mortgage = 2,100,000
Thus, the total interest is the difference between the total amount paid and the
Definition of Terms amount of mortgage:
Amortization Method – method of paying a loan (principal and interest) on Total interest = 2,100,000 − 1,500,000
installment basis, usually of equal amounts at regular intervals. = 600,000.
Mortgage – a loan, secured by a collateral, that the borrower is obliged to pay at Example 14. Emelyn wants to purchase a house that is being sold for P2,500,000.
specified terms. The bank requires 25% down payment, find the amount of the mortgage.
Chattel Mortgage – a mortgage on a movable property. Solution.
Collateral – assets used to secure the loan. It may be a real-estate or other Down payment = down payment rate × cash price
investments. = (0.25%)(2,500,000)
Outstanding Balance – any remaining debt at a specified time. = 625,000.
Amount of the loan = cash price – down payment
= 2,500,000 − 625,000

23
ASIAN INSTITUTE OF COMPUTER STUDIES (AICS)

= 1,875,000
The mortgage amount is P1,875,000. How to read a Stock Table
https://www.dummies.com/personal-finance/investing/how-to-read-a-
Example 15. Mr. Errol bought a van for his business. After paying the down
stock-table/
payment, the amount of loan is P700,000 with an interest rate of 6% compounded
monthly. The term of the loan is 4 years. How much is the monthly payment?
Stock Index Table, July 10,2020 Philippine Stock Exchange Index (PSEi)
Given.
https://www.pse.com.ph/stockMarket/home.html
𝑃 = 700,000
𝑖 (12) = 0.06
𝑖 (12) 0.06
𝑗= = = 0.005
12 12
Find the Regular Payment 𝑹.
Solution.
𝑃 700,000
𝑅= 1−(1+𝑗)−𝑛 = 1−(1+0.005)−48
= 16,439.5203
[ ] [ ]
𝑛 0.005
The regular payment is P16,439.52.

REFERENCES
Books
Department of Education, (2016) General Mathematics Learner’s Material
First Edition 2016, Lesson 31-35, pp. 212-244

Vibal Inc., Chan, J.H.N., Miro, E.D., Quiming, R.S., (2016) General
Mathematics; Lesson 2.4 Consumer Loans; Lesson 2.5 Business Loans;
Lesson 2.6 Bonds; Lesson 2.7 Stocks, pp. 89-133

Electronic Sources

Hayes, A. Updated Feb. 22, 2020: Bond retrieved from


https://www.investopedia.com/terms/b/bond.asp

Hayes, A. Updated Feb. 25, 2020: Preferred vs. Common Stock: What’s
the difference? retrieved from
https://www.investopedia.com/ask/answers/difference-between-
preferred-stock-and-common-stock/

24
ASIAN INSTITUTE OF COMPUTER STUDIES (AICS)

Activity 1: Stuck in Stocks and Bonds I

Directions. Solve the following problems involving stocks and bonds. Answers are 3. A bank declared a dividend of P27 per share for the common stock. If the
provided in the stocks provided below. Still, show your complete solution. Answers common stock close at P93, how large is the stock yield ratio on this
without solution is considered wrong. investment?

0.35 P1,500 P83.33


4. Find the amount of the semi-annual coupon for a P200,000 bond which
pays 5% convertible semi-annually for its coupons.

P138.89 0.29
5. Determine the amount of semi-annual coupon paid for a 3% bond with a
face value of P100,000 which matures after 8 years. How many coupons
are paid?
P5,000
1. A financial institution declared a dividend of P75,000,000 for its common
6. A food corporation declared a dividend P25,000,000 for its common stock.
stock. Suppose there are 900,000 shares of common stock, how much is
Suppose there are 180,000 shares of common stock, how much is the
the dividend per share?
dividend per share?

2. The ABC corporation gave out P38 dividend per share for its common stock.
The market value of the stock is P108. Determine the stock yield ratio.

25
ASIAN INSTITUTE OF COMPUTER STUDIES (AICS)

Activity 2: Stuck in Stocks and Bonds II Activity 3: Too Personal or Business Only?

Directions. Consider the following listing on stocks and answer the questions that Directions. Identify whether the following illustrates a business loan or a costumer
follow: loan.
52-WK 52-WK STOCK DIV YLD% VOL CLOSE NETCHG 1. On his father’s 50th birthday, Jade decided to take her family for a vacation.
HI LOW (100s) To cover the expenses, he decided to apply for a loan.
75 65 JJJ 2.5 2.8 1500 70 2 2. Aubrey was just approved of a loan for her to buy a new phone.
34 23 KKK 1.7 1.75 1200 28 -3 3. Ms. Gallardo wants to expand her laundry business. She decided to apply
for a loan to buy new laundry appliances.
For Stock JJJ:
4. Carlo wishes to borrow money for the purchase of a new LED TV worth
1. What was the highest price of the stock for the last 52 weeks?
P45,000.
2. What was the dividend per share last year?
5. Mr. Rasos wants to have another branch for his tutorial center business.
3. What was the annual percentage yield last year?
He decided to apply for a loan that he can use to pay for the rentals of the
4. What was the closing price in the last trading day?
new branch.
5. What was the closing price the day before the last trading day?
6. Victor decided to purchase a condominium unit near his workplace. He got
For Stock KKK? a loan worth P1,500,000.
6. What was the lowest price of the stock for the last 52 weeks? 7. Mrs. Malay runs a trucking business. She wants to buy three more trucks
7. How many shares were traded in the last trading day? for expansion of her business. She applied for a loan in a bank.
8. What was the dividend per share? 8. Mr. Ching wants to buy materials needed for his gadgets store, but these
9. What was the closing price in the last trading day? materials are only available abroad. He applied for a loan in a bank in order
10. What was the closing price in the day before the last trading day? to afford these goods.
9. Christine wants to fix her dental problems. She borrowed money in a bank
to afford dental braces.
10. The online business of Marian was a success, so she decided to provide her
own courier service. Her loan was just approved. With the money she had,
she bought a new motorcycle she can use in delivering products.

26
ASIAN INSTITUTE OF COMPUTER STUDIES (AICS)

Activity 4: All About Loans

Directions. Solve the following problems involving loans. Show you complete
5. You have just graduated from college and landed your first big job. You
solution.
have always dreamed of being a homeowner, and after carefully shopping
for your dream home, you find one that you would like to purchase at a
1. A loan of P300,00 is to be repaid in full after 2 years. If the interest rate is
cost of P250.000. After researching banks to find the best interest rate,
9% per annum, how much should be paid after 2 years?
you find that Banks for Homeowners offers the best rate of 6% interest
that compounds monthly for 30 years. What is the monthly payment for
this loan?

2. If a car loan of P790,000 requires a 20% down payment. How much is the
mortgage?

3. A person borrowed P1,000,000 for the purchase of a car. If his monthly


payment P25,000 on a 8-year mortgage, find the total amount of interest.

4. A business loan worth P250,000 is to repay in quarterly installment in 1


year. How much is the quarterly payment if money is worth 8% converted
quarterly?

27
ASIAN INSTITUTE OF COMPUTER STUDIES (AICS)

CHAPTER 3.1 LOGIC 


KEY QUESTIONS
How do you construct a
2nd QUARTER
MODULE 11

At the end of this module, you are expected to: truth table?
WEEK _____

 Illustrate, symbolize, and perform operations on propositions,  What are the logical
 Determine the truth values of propositions, operators for?
 Interpret different types of tautologies and fallacies, and  How can two propositions
 Familiarize logical operators and its difference to each other. be logically equivalent?

𝑝1 : It Is not the case that √2 is a rational number.


3.1.1 Propositions
𝑝2 : Either logic is fun and interesting, or it is boring.
𝑝3 : If you are a Grade 11 student, then you are a Filipino.
A proposition is a declarative sentence that is either true or false, but not both.
𝑝4 : If you are more than 60 years old, then you are entitled to a Senior Citizen’s
If a proposition is true, then its truth value is true, which is denoted by 𝑇,
card, and if you are entitled to a Senior Citizen’s card, then you are more than 60
otherwise, its truth value is false, which is denoted by 𝐹.
years old.
We use small letters like 𝑝. 𝑞. 𝑟 to denote propositions. When several statements
are involved, we use 𝑝1 , 𝑝2 , … , 𝑝𝑛 . For example, the proposition: Solution. Recall that for a statement to be a proposition, it must be a declarative
sentence, and it should have a truth value of either true or false, but not both.
𝑝: Everyone should study logic
May be read as: 𝑝: This is a declarative sentence, and since Tacloban is a city in the province of
“p is the proposition: Everyone should study logic.” Leyte, which is in Visayas, hence 𝑝 is a proposition.

Example 1. Determine whether each of the following statements is a proposition 𝑞: This is an imperative sentence, so it is not a proposition.
or not. If it is a proposition, give its truth value.
𝑟: This statement is a declarative sentence, but its truth value will only be known
𝑝: Tacloban is a city in Visayas. after the logic exam. Nonetheless, it can either be true or false, but not both. Hence
𝑞: Find a color that suits your taste. 𝑟 is a proposition.
𝑟: My seatmate will get a perfect score in the logic exam.
𝑠: This statement is exclamatory, so it is not a proposition.
𝑠: Mabuhay!
𝑡: 1 + 2 = 3 𝑡: The given equations is a mathematical sentence. Translated into English: the
𝑢: 𝑓(𝑥) =
√𝑥
is a rational function equation reads “the sum of one and two is three”, which is a declarative sentence.
𝑥−2
It is also a true statement. Hence, 𝑡 is a true proposition.
𝑣: What is the range of the function?
𝑤: I am lying.

28
ASIAN INSTITUTE OF COMPUTER STUDIES (AICS)

𝑢: This is a declarative statement. Since the numerator of the function is not a Example 2. For each of the propositions in Example 1, determine whether it is a
polynomial function, the function 𝑓 is not a rational, so the statement is false. It is simple or a compound proposition. If it is a compound proposition, identify the
therefore a false proposition. simple components.

𝑣: This is an interrogative sentence. Hence it is not a proposition. Solution. The propositions 𝑝, 𝑟, 𝑡, 𝑢, 𝑣, and 𝑤 are all simple propositions. On the
𝑤: Although 𝑤 is a declarative sentence, it is not a proposition because it can other hand, the following are compound propositions:
neither be true or false. Suppose 𝑤 is true─ that is, in case that I am lying. Since I
𝑝1 : It is not the case that √2 is a rational number.
am lying, my statement is not true, and so 𝑤 must be false. The same conclusion
can be drawn if 𝑤 is assumed to be false. 𝑝2 : Either logic is fun and interesting, or it is boring.

𝑝1 : While this is declarative sentence, it can be shown that √2 cannot be expressed 𝑝3 : If you are a Grade 11 student, then you are a Filipino.
as a quotient of two integers. Thus 𝑝1 is false proposition.
𝑝4 : If you are more than 60 years old, then you are entitled to a Senior Citizen’s
𝑝2 : This is a declarative sentence, which is true since students may find logic either card, and if you are entitled to a Senior Citizen’s card, then you are more than 60
fun or boring. Hence 𝑝2 is a true proposition. years old.

𝑝3 : This is a declarative sentence, but it is not true. There are also Grade 11 Furthermore, we can determine the simple propositions that make up the
students of other nationalities. propositions 𝑝1, 𝑝2 , 𝑝3 and 𝑝4 . We do so in the following table:

𝑝4 : We know that it is a true proposition. This is also known as a biconditional Proposition Simple Component/s
statement, since we can rewrite it as “You are more than 60 years old if and only if 𝑝1 𝑎: √2 is not a rational number
you are entitled to a Senior Citizen’s card.” 𝑝2 𝑏: Logic is fun
𝑐: Logic is interesting
A compound proposition is a proposition formed from simpler proposition using 𝑑: Logic is boring
logical connectors or some combination of logical connectors. Some logical 𝑝3 𝑒: You are a Grade 11 student
𝑓: You are a Filipino
connectors involving propositions 𝑝 and/or 𝑞 may be expressed as follows:
𝑝4 𝑔: You are more than 60 years old
not p
ℎ: You are entitled to a Senior Citizen’s
p and q card
p or q
If p, then q The compound proposition can thus be expressed as follows:
where <. > stands for some proposition. A proposition is simple if it cannot be 𝑝1: not 𝑎
broken down any further into other component propositions. 𝑝2 : 𝑎 and 𝑏 or 𝑐
𝑝3 : if 𝑒, then 𝑓
𝑝4 : (if 𝑔 then ℎ) and (if ℎ then 𝑔)

29
ASIAN INSTITUTE OF COMPUTER STUDIES (AICS)

Negation
3.1.2 Logical Operators
Let 𝑝 be a proposition. The negation of 𝒑 is the proposition
“It is not true that 𝑝.”
One of the most useful tools in propositional logic is the truth table. It is used when
It is denoted by ~𝑝, which read as not 𝒑.
we have a proposition whose primitives (i.e., simple components) are abstract
propositions, meaning, we do not know nor care what statement they for, but we It is then defined through its truth table
know that they can only either true (T) or false (F). We will now discuss how to
𝑝 ~𝑝
construct it. T F
In the next figure, all the possible truth values of one, two, and three propositions F T
are presented in their respective truth tables. Suppose 𝑝, 𝑞, and 𝑟 are (primitive)
propositions. Since each of the propositions 𝑝 and 𝑞 has two possible truth values, Table 3.b
there are four (= 22 ) possible combinations of truth values 𝑝 and 𝑞. For three Example 3. State the negation of the following propositions.
propositions 𝑝, 𝑞, and 𝑟, there are eight = (22 ) possible truth combinations. In 𝑛1 : 𝑝(𝑥) =
𝑥−2
is a polynomial function.
𝑥+2
general, the truth table involving 𝑛 propositions would have 𝑛 columns and
𝑛2 : 1 is an even number.
2𝑛 rows; the rows correspond to each of the possible 2𝑛 possible combinations of
𝑛3 : The tinikling is the most difficult dance.
truth values for 𝑛 propositions.
𝑛4 : Everyone in Visayas speaks Cebuano.
𝑝 𝑞 𝑝 𝑞 𝑝 𝑞 𝑟
T T T T T T T Solution.
F F T F T T F 𝑥−2
F T T F T 𝑛1 : “It is not true that 𝑝(𝑥) = 𝑥+2 is a polynomial function” or simply
T F F 𝑥−2
F F “𝑝(𝑥) = 𝑥+2 is not a polynomial function”.
F T T
F T F 𝑛2 : “It is not true that 1 is an even number”, or “1 is an odd number”.
F F T
F F F 𝑛3 : “The tinikling is not the most difficult dance”.

𝑛4 : “Not everyone in the Visayas speaks Cebuano”.

Conjunction
Table 3.a
Let 𝑝 and 𝑞 be propositions. The conjunction of the propositions 𝑝 and 𝑞 is the
A logical operator (or connective) on mathematical statements is a word or
proposition
combination of words that combines one or more mathematical statements to
“𝑝 and 𝑞,”
make a new mathematical statement. (Sundstrom, 2019)
Which is denoted by 𝑝 ∧ 𝑞 is true if both 𝑝 and 𝑞 are true; otherwise, 𝑝 ∧ 𝑞 is
false. The proposition 𝑝 and 𝑞 are called the conjuncts of 𝒑 ∧ 𝒒.

30
ASIAN INSTITUTE OF COMPUTER STUDIES (AICS)

It is then defined through its truth table It is then defined through its truth table

𝑝 𝑞 𝑝∧𝑞 𝑝 𝑞 𝑝∨𝑞
T T T T T T
T F F T F T
F T F F T T
F F F F F F

Table 3.c Table 3.d


Example 4. Let 𝑝 and 𝑞 be the following propositions: The above truth table shows us that the disjunction 𝑝 ∨ 𝑞 is false only when both
𝑝: Today is my birthday disjuncts are false.
𝑞: I am happy Example 5. Let 𝑝, 𝑞, and 𝑟 be the following propositions:
𝑝: Joy and Mark will clean the house.
Express the following conjunctions in English sentences or in symbols, as the case 𝑞: Jade is sleeping.
may be. 𝑟: Emhar is eating.
1. 𝑝 ∧ 𝑞
2. 𝑝 ∧ (~𝑞) Express the following propositions in English sentences or in symbols, as the case
3. “Today is not my birthday and I am not happy” may be.
4. “While today is not my birthday, I am happy” 1. 𝑝 ∨ 𝑞
2. 𝑞 ∨ (~𝑟)
Solution. 3. 𝑝 ∨ (𝑞 ∨ 𝑟)
1. Today is my birthday and I am happy. 4. “Either Joy and Mark will clean the house or Jade is sleeping, or Emhar is
2. Today is my birthday and I am not happy. eating.”
3. (~𝑝) ∧ (~𝑞) 5. “Either Joy and Mark will clean the house and Jade is sleeping, or Emhar is
4. (~𝑝) ∧ 𝑞 eating.”
6. “Either Joy and Mark will clean the house or Jade is sleeping, and Emhar is
Disjunction eating.”
Let 𝑝 and 𝑞 be propositions. The disconjunction of the propositions 𝑝 and 𝑞 is 7. “Either Joy and Mark will clean the house and Jade is sleeping, or Joy and
the proposition Mark will clean the house and Emhar is eating.”
“𝑝 or 𝑞,” Solution.
which is denoted by 𝑝 ∨ 𝑞. The disjunction 𝑝 ∨ 𝑞 is true only when at least one 1. Joy and Mark will clean the house or Jade is sleeping.
𝑝 and 𝑞 is true; otherwise, 𝑝 ∨ 𝑞 is false. The propositions 𝑝 and 𝑞 are called the 2. Either Jade is sleeping or Emhar is not eating.
disjuncts of 𝒑 ∨ 𝒒.

31
ASIAN INSTITUTE OF COMPUTER STUDIES (AICS)

3. Either Joy and Mark will clean the house, or Jade is sleeping, or Emhar is Clearly, 𝑝 is the hypothesis, while 𝑞 is the conclusion. Then 𝑟, in symbols, is 𝑝 → 𝑞.
eating. This implication will be true if I keep my promise and false if I fail to do so.
4. (𝑝 ∨ 𝑞) ∨ 𝑟 Suppose you did wash the dishes (𝑝 is true) and I give you 20 pesos (𝑞 is true). Since
5. (𝑝 ∧ 𝑞) ∨ 𝑟 this scenario means I kept my promise, the implication is true. This corresponds to
6. 𝑝 ∨ (𝑞 ∧ 𝑟) the first row in the figure given above.
7. (𝑝 ∧ 𝑞) ∨ (𝑝 ∧ 𝑟) Suppose you washed the dishes (𝑝 is true), but I gave you 100 pesos (𝑞 is false).
Since I did not give you 20 pesos as promised, the implication is false. The scenario
Conditional
corresponds to the second row in the figure given above.
Let 𝑝 and 𝑞 be propositions. The conditional of the propositions 𝑝 and 𝑞 is the Now, suppose that you did not wash the dishes (𝑝 is false). Whether I give you the
proposition promised amount (𝑞 is true) or not (𝑞 is false) does not matter, the promise is
“if 𝑝 then 𝑞,” considered fulfilled. Hence, in both cases, the implication is true. These cases
which is false when 𝑝 is true and 𝑞 is false, and true otherwise. It is written correspond to the last two rows in the figure above.
symbolically as 𝑝 → 𝑞, which can also be read as “𝑝 implies 𝑞”. In this
implication, 𝑝 is called the hypothesis and 𝑞 is called the conclusion. Biconditional
Let 𝑝 and 𝑞 be propositions. The biconditional of the propositions 𝑝 and 𝑞 is the
It is then defined through its truth table: proposition
“if 𝑝 then 𝑞,”
𝑝 𝑞 𝑝→𝑞
T T T which is true when 𝑝 and 𝑞 have the same truth values, and false otherwise. It is
T F F written symbolically as 𝑝 ↔ 𝑞.
F T T
F F T It is then defined through its truth table:

Table 3.e 𝑝 𝑞 𝑝↔𝑞


T T T
Example 6. Consider the conditional, T F F
F T F
𝑝: If you wash the dishes, then I will give you 20 pesos. F F T
Now let 𝑝 and 𝑞 be the following primitives of the implication:
Table 3.f
𝑝: You wash the dishes
and The proposition may also be written as “𝑝 iff 𝑞”. The propositions 𝑝 and 𝑞 are the
𝑞: I give you 20 pesos same components of the biconditional.

32
ASIAN INSTITUTE OF COMPUTER STUDIES (AICS)

Example 7. Suppose that Fiona is a Grade 11 student. Let us now consider the Solution. Note that there are two propositions, 𝑝 and 𝑞, involved in the compound
following biconditionals. proposition. Thus, the truth table will contain 4 rows, the first two columns of
𝑝1 : Fiona is in Grade 11 if and only if she is a senior high school student. which are
𝑝2 : Fiona is in Grade 11 if and only if she is working as a lawyer. 𝑝 𝑞
𝑝3 : Fiona has a degree in Computer Science if and only if she believes in true love. T T
T F
Solution. F T
𝑝1 : Again, both simple components of 𝑝1 are true. Therefore, the biconditional F F
statement is true. Using the truth table for the definition of the conditional statements 𝑝 → 𝑞 and
𝑝2 : It is true that Fiona is in Grade 11, but it is not true that Fiona is working as a 𝑞 → 𝑝, we add two more columns to indicate the truth values of 𝑝 → 𝑞 and 𝑞 → 𝑝.
lawyer. Therefore, the biconditional is not true, referring to the second row of its
truth table. 𝑝 𝑞 𝑝→𝑞 𝑞→𝑝
𝑝3 : The first simple proposition, that Fiona has a degree in Computer Science is T T T T
false. The truth value of the entire biconditional depends on the truth value of the T F F T
second simple component, that she believes in true love. If this is true, then the F T T F
biconditional is false (referring to the third row of the truth table); otherwise, the F F T T
biconditional is true.
In the final column, we add the truth value of (𝑝 → 𝑞) ∧ (𝑞 → 𝑝), which is a
In the next table, we show that when both the implication 𝑝 → 𝑞 and 𝑞 → 𝑝 are conjunction involving 𝑝 → 𝑞 and 𝑞 → 𝑝 as disjuncts.
true, the biconditional 𝑝 ↔ 𝑞 is true. This is the reason why we use the notation
𝑝 𝑞 𝑝→𝑞 𝑞→𝑝 (𝑝 → 𝑞) ∧ (𝑞 → 𝑝)
𝑝 ↔ 𝑞 and read it as “𝑝 if and only if 𝑞”, which is also commonly written as “𝑝 if 𝑞”. T T T T T
There are several grammatical constructions for the biconditional, which are given T F F T F
below. F T T F F
F F T T T
𝑝 is necessary and sufficient for 𝑞
If 𝑝 then 𝑞, and conversely. Example 9. Consider the compound proposition [(𝑝 → 𝑟) ∧ (𝑞 → 𝑟)] → [(𝑝 ∨
𝑝 is equivalent to 𝑞 𝑞) → 𝑟]. Construct its table.
𝑝 iff 𝑞
Solution. There are three primitive propositions involved, and so the truth table
3.1.3 Truth Tables for the compound proposition has 8 rows.
We first consider the truth table pertaining to (𝑝 → 𝑟) ∧ (𝑞 → 𝑟), the hypothesis
Example 8. Let 𝑝 and 𝑞 be propositions. Construct the truth table for the compound of the entire conditional.
proposition (𝑝 → 𝑞) ∧ (𝑞 → 𝑝).

33
ASIAN INSTITUTE OF COMPUTER STUDIES (AICS)

F T T T T T T T T
𝑝 𝑞 𝑟 𝑝→𝑟 𝑞→𝑟 (𝑝 → 𝑟) ∧ (𝑞 → 𝑟) F T F T F F T F T
T T T T T T F F T T T T F T T
T T F F F F F F F T T T F T T
T F T T T T
T F F F T F Note the regardless of the truth tables of 𝑝, 𝑞, and 𝑟, the proposition 𝑠 is always
F T T T T T true. Such propositions are called tautologies.
F T F T F F
F F T T T T A proposition that is always true is called a tautology, while a proposition that is
F F F T T T always false is called a contradiction. We denote tautologies by 𝜏 and
contradictions by ∅. A proposition that is neither a tautology nor a contradiction
Next, we consider the conclusion (𝑝 ∨ 𝑞) → 𝑟 of the conditional. For this, we
is called a contingency.
require the truth table of 𝑝 ∨ 𝑞 and (𝑝 ∨ 𝑞) → 𝑟, which will be appended to the
table above.
3.1.4 Logical Equivalence and Conditional Propositions
𝑝 𝑞 𝑟 𝑝→𝑟 𝑞→𝑟 (𝑝 → 𝑟) ∧ (𝑞 → 𝑟) 𝑝∨𝑞 (𝑝 ∨ 𝑞) → 𝑟
T T T T T T T T
T T F F F F T F Two propositions 𝑝 and 𝑞 are logically equivalent, denoted by 𝑝 ⇔ 𝑞, if they
T F T T T T T T have the same truth values for all possible truth values of their simple (primitive)
T F F F T F T F components.
F T T T T T T T
F T F T F F T F Example 10. Show that (𝑝 → 𝑞) ⇔ [(~𝑝) ∨ 𝑞].
F F T T T T F T Solution.
F F F T T T F T 𝑝 𝑞 𝒑 → 𝒒 ~𝑝 (~𝒑) ∨ 𝒒
T T T F T
Lastly, we consider the truth value of the proposition 𝑠 which we define to be T F F F F
[(𝑝 → 𝑟) ∧ (𝑞 → 𝑟)] → [(𝑝 ∨ 𝑞) → 𝑟] F T T T T
F F T T T
𝑝 𝑞 𝑟 𝑝 𝑞→𝑟 (𝑝 → 𝑟) ∧ (𝑞 → 𝑝∨𝑞 (𝑝 ∨ 𝑞) 𝑠
→𝑟 𝑟) →𝑟
T T T T T T T T T Same values
T T F F F F T F T Notice that the third and fifth column above contains the same truth values in the
T F T T T T T T T same sequence. Thus, (𝑝 → 𝑞) ⇔ [(~𝑝) ∨ 𝑞]. We call this logical equivalence the
T F F F T F T F T Switcheroo law.

34
ASIAN INSTITUTE OF COMPUTER STUDIES (AICS)

The Switcheroo law is the logical equivalence 𝑝 ↔ 𝑞 ⇔ [(𝑝 → 𝑞) ∧ (𝑞 → 𝑝)]


(𝑝 → 𝑞) ⇔ (~𝑝) ∨ 𝑞
Example 11. Show that ~(𝑝 → 𝑞) ⇔ [𝑝 ∧ (~𝑞)] using logical equivalences.
In words, it expresses the equivalence between saying “if 𝑝 is true, then 𝑞 must
Solution.
be true” and saying “either 𝑝 is not true, or else 𝑞 must be true”.
Reason
The table below shows the different logical equivalences that are used when
~(𝑝 → 𝑞) Given
manipulating compound propositions.
⇔ ~((~𝑝) ∨ 𝑞) Switcheroo
⇔ ~(~𝑝) ∧ (~𝑞) De Morgan’s Law
Theorem (Table of Logical Equivalences)
⇔ 𝑝 ∧ (~𝑞) Double Negation
Let 𝑝, 𝑞, and 𝑟 be propositions. We have the following logical equivalences.
Example 12. Let 𝑝 and 𝑞 be propositions. Construct the truth tables for each of the
Identity Laws (𝑝 ∧ 𝜏) ⇔ 𝑝 (𝑝 ∨ ∅ ) ⇔ 𝑝
(𝑝 ∨ 𝜏) ⇔ 𝜏 (𝑝 ∧ ∅) ⇔ ∅ following conditionals: 𝑝 → 𝑞, 𝑞 → 𝑝, (~𝑝) → (~𝑞), (~𝑞) → (~𝑝)
Domination
Laws Solution. We construct a single truth table containing each of the conditionals:
Idempotent (𝑝 ∨ 𝑝) ⇔ 𝑝 (𝑝 ∧ 𝑝) ⇔ 𝑝 𝑝 𝑞 ~𝑝 ~𝑞 𝑝→𝑞 𝑞→𝑝 (~𝑝) → (~𝑞) (~𝑞) → (~𝑝)
Laws T T F F T T T T
Inverse Laws [𝑝 ∨ (~𝑝)] ⇔ 𝜏 [𝑝 ∧ (~𝑝)] ⇔ ∅ T F F T F T T F
Double ~(~𝑝) ⇔ 𝑝 F T T F T F F T
Negation F F T T T T T T
Associative 𝑝 ∨ (𝑞 ∨ 𝑟) ⇔ (𝑝 ∨ 𝑞) ∨ 𝑟 𝑝 ∧ (𝑞 ∧ 𝑟) ⇔ (𝑝 ∧ 𝑞) ∧ 𝑟
Laws
Commutative 𝑝∨𝑞 ⇔𝑞∨𝑝 𝑝∧𝑞 ⇔𝑞∧𝑝 Same values
Laws Note that the fifth and eight columns are the same, so we have shown that
Distributive 𝑝 ∨ (𝑞 ∧ 𝑟) ⇔ (𝑝 ∨ 𝑞) ∧ (𝑝 ∨ 𝑟) 𝑝 ∧ (𝑞 ∨ 𝑟) ⇔ (𝑝 ∧ 𝑞) ∨ (𝑝 (𝑝 → 𝑞) ⇔ [(~𝑞) → (~𝑝)]. Likewise, since the sixth and seventh columns are
Laws ∧ 𝑟)
identical, if follows that (𝑞 → 𝑝) ⇔ [(~𝑝) → (~𝑞)].
De Morgan’s ~(𝑝 ∨ 𝑞) ⇔ (~𝑝) ∧ (~𝑞) ~(𝑝 ∧ 𝑞) ⇔ (~𝑝) ∨ (~𝑞) The conditionals that we considered in the previous examples are the different
Laws
forms of conditional propositions.
Absorption 𝑝 ∨ (𝑝 ∧ 𝑞) ⇔ 𝑝 𝑝 ∧ (𝑝 ∨ 𝑞) ⇔ 𝑝
Laws Definition. Suppose that 𝑝 and 𝑞 are propositions. From the conditional
proposition 𝑝 → 𝑞, we derive three other conditional statements, namely its:
Table 3.g
a. Converse: 𝑞 → 𝑝
Note: In the previous lecture, it was shown that 𝑝 ↔ 𝑞 has the same truth table b. Contrapositive: (~𝑞) → (~𝑝)
as (𝑝 → 𝑞) ∧ (𝑞 → 𝑝). Therefore, we can say that: c. Inverse: (~𝑝) → (~𝑞)

35
ASIAN INSTITUTE OF COMPUTER STUDIES (AICS)

Example 13. Let the proportions 𝑝 and 𝑞 be 𝑥 not equal to 2 and −2, the hypothesis 𝑞: 𝑥 2 − 4 = 0 is false. Again, by the last
two rows of Table 3.e, the implication 𝑞 → 𝑝 is true. It is also true for 𝑥 = −2, since
𝑝: 𝑥 − 2 and 𝑞: 𝑥 2 − 4 = 0
both the hypothesis and conclusion are true. However, when 𝑥 = 2, hypothesis
The implication 𝑝 → 𝑞 is true when 𝑥 = −2 by the first row of Table 3.e because 𝑥 2 − 4 = 0 is also true but the conclusion 𝑥 = −2 is obviously false. By the second
when 𝑥 = −2 the hypothesis 𝑝: 𝑥 − 2 is true and the conclusion 𝑞: 𝑥 2 − 4 = 0 is row of Table 3.e, the implication 𝑞 → 𝑝 is false for 𝑥 = 2. Thus, the implication 𝑞 →
also true. For the case when 𝑥 ≠ −2, the hypothesis 𝑝: 𝑥 − 2 is false. According to 𝑝 is true for all real number 𝑥 except when 𝑥 = 2.
the last two rows of Table 3.e, regardless of whether the conclusion 𝑞: 𝑥 2 − 4 = 0
Another implication which can be derived from 𝑝 → 𝑞 is its inverse ~𝑝 → ~𝑞. In
is true or false, the implication 𝑝 → 𝑞 is true. Hence, the implication 𝑝 → 𝑞 is true
the example above, the inverse of the proposition 𝑝 → 𝑞 is the implication
for any real number 𝑥.
~𝑝 → ~𝑞: If 𝑥 ≠ −2, then 𝑥 2 − 4 = 0
𝑝 𝑞 𝑝→𝑞
T T T It can also be shown that ~𝑝 → ~𝑞 is true for all real numbers 𝑥, except when 𝑥 =
T F F 2. We summarize these findings in the next table.
F T T
F F T Conditional 𝑝→𝑞 If 𝑥 = −2, then True for any real
Table 3.e (Implication) 𝑥2 − 4 = 0 number 𝑥
Contrapositive ~𝑞 → ~𝑝 If 𝑥 2 − 4 ≠ 0, True for any real
The negation of the open propositions then 𝑥 ≠ −2 number 𝑥
Converse 𝑞→𝑝 If 𝑥 2 − 4 = 0, True for all real
𝑝: 𝑥 − 2 and 𝑞: 𝑥 2 − 4 = 0 number 𝑥 ≠ 2
then 𝑥 = −2
are, respectively, the open propositions Inverse ~𝑝 → ~𝑞 If 𝑥 ≠ −2, then True for all real
𝑥2 − 4 ≠ 0 number 𝑥 ≠ 2
~𝑝: 𝑥 ≠ −2 and ~𝑞: 𝑥 2 − 4 ≠ 0

In general, the implication ~𝑞 → ~𝑝 is called the contrapositive of the implication In the table above, we see that the 𝑝 → 𝑞 and its contrapositive ~𝑞 → ~𝑝 have the
𝑝 → 𝑞. In this example, the contrapositive of 𝑝 → 𝑞 is the proposition same truth values, and its converse and its inverse also have the same truth values.
(~𝑞) → (~𝑝): If 𝑥 2 − 4 ≠ 0, then 𝑥 ≠ −2 This is not a mere coincidence as shown in the table given in Example 12.

Example 14. Let 𝑝 → 𝑞 be the political slogan: “If there are no corrupt people, then
Similarly, it can be verified that the contrapositive ~𝑞 → ~𝑝 is also true for any real
there are no poor people.” State the converse, contrapositive, and the inverse of
number 𝑥. Now consider the proposition 𝑞 → 𝑝:
𝑝 → 𝑞.
If 𝑥 2 − 4 = 0, then 𝑥 = −2 Solution.

The implication 𝑞 → 𝑝 is called the converse of 𝑝 → 𝑞. Recall that the solutions of Converse (𝑞 → 𝑝): “If there are no poor people, then there are no corrupt people.”
the quadratic equation 𝑥 2 − 4 = 0 are 𝑥 = −2 and 𝑥 = 2. So for any real number

36
ASIAN INSTITUTE OF COMPUTER STUDIES (AICS)

Contrapositive ((~𝑞) → (~𝑝)): “If there are poor people, then there are corrupt
people.”

Inverse ((~𝑝) → (~𝑞)): “If there are corrupt people, then there are poor people.”

REFERENCES
Books
Department of Education, (2016) General Mathematics Learner’s Material
First Edition 2016, Lesson 36-41, pp. 244-295

Vibal Inc., Chan, J.H.N., Miro, E.D., Quiming, R.S., (2016) General
Mathematics; Lesson 3.1 Propositions and Connectives; Lesson 3.2 More
on Conditionals and Truth Tables; 3.3 Rules of Inference, pp. 158-183

Electronic Sources

Sundstrom, T. Updated Jun 6, 2019: Statements and Logical Operators


retrieved from
https://math.libretexts.org/Bookshelves/Mathematical_Logic_and_Proof
/

Logical Operators
https://images.app.goo.gl/AfufSUy7phxihc1f6

Waner, S. Costenoble, Steven R. Introduction to Logic (3. The Conditional


and the Biconditional) retrieved from
https://www.zweigmedia.com/RealWorld/logic/logic3.html

37
ASIAN INSTITUTE OF COMPUTER STUDIES (AICS)

Activity 1: Proposition or Not? Activity 2: Being Negative

Directions. Determine whether the following statements are propositions. If it is, Directions. State the negation of the following propositions.
write it under the “Propositions” column, and if not, write it under “Not a
Proposition”. 1. Christine received a
2. 𝑥 2 + 𝑦 2 = 𝑧 2 .
handwritten letter for her friend.
Proposition Not a Proposition ____________________________
____________________________
____________________________
____________________________
____________________________
____________________________

3. Kimberly teaches high-level 4. Mark was not able to fetch his


1. Please knock on the door before you come in. mathematics to college students. sister from school today.
2. If a parallelogram has opposite equal acute angles, opposite equal obtuse ____________________________ ____________________________
angles, and four equal sides, then it is a rhombus. ____________________________ ____________________________
3. Don’t ever touch my phone. ____________________________ ____________________________
4. 169 is a perfect square.
5. 2 + 3 = 5.
6. Either I take a master’s degree next year, or I will renew my contract.
5. The student brings at most two
7. Did you exercise this morning?
bags with him every school day.
8. Prince Zuko is a firebender and the equation 𝑥 2 + 1 has no real solutions.
____________________________
9. Go straight ahead then turn left.
____________________________
10. Wow! That is great news!
____________________________
11. Rizal is the capital of the Philippines.
12. This sentence is false.

38
ASIAN INSTITUTE OF COMPUTER STUDIES (AICS)

Conditional 4. “Yachi studied for the exam alone if her boyfriend decided
Activity 3: Conditional Transformation 𝑝→𝑞 to go home early to sleep.”
Converse
𝑞→𝑝
Directions. Determine the converse, contrapositive and the inverse of the Contrapositive
following conditional propositions. (~𝑞) → (~𝑝)
Inverse
Conditional 1. “If it is consumed in large volumes, then chocolate can be
(~𝑝) → (~𝑞)
𝑝→𝑞 harmful to one’s health.”
Converse
𝑞→𝑝 Conditional 5. “If at least one student slept in class, then Kiyoko gave a
Contrapositive
𝑝→𝑞 difficult quiz to wake the students.”
(~𝑞) → (~𝑝)
Converse
Inverse
𝑞→𝑝
(~𝑝) → (~𝑞)
Contrapositive
(~𝑞) → (~𝑝)
Inverse
Conditional 2. “Whenever he will be given the chance to perform on stage,
(~𝑝) → (~𝑞)
𝑝→𝑞 Asahi will sing all his classic songs.”
Converse
𝑞→𝑝
Contrapositive Conditional 6. “Whenever Kageyama studied alone, he got the highest
(~𝑞) → (~𝑝) 𝑝→𝑞 score in the class.”
Inverse Converse
(~𝑝) → (~𝑞) 𝑞→𝑝
Contrapositive
(~𝑞) → (~𝑝)
Conditional 3. “If Shoyo will decide to enter through the front door, then Inverse
𝑝→𝑞 he will be greeted by a large group of press people.” (~𝑝) → (~𝑞)
Converse
𝑞→𝑝
Contrapositive
(~𝑞) → (~𝑝)
Inverse
(~𝑝) → (~𝑞)

39
ASIAN INSTITUTE OF COMPUTER STUDIES (AICS)

Activity 4: Truth Tables


4. (𝑝 → 𝑞) → (𝑞 → 𝑝)

𝑝 𝑞 𝑝→𝑞 𝑞→𝑝 (𝑝 → 𝑞) → (𝑞 → 𝑝)
Directions. Complete each truth table. Conclude if each table is a tautology,
T T
contradiction, or contingency.
T F
F T
1. (𝑝 ∧ 𝑞) → (𝑝 ∨ 𝑞) F F
𝑝 𝑞 𝑝∧𝑞 𝑝∨𝑞 (𝑝 ∧ 𝑞) → (𝑝 ∨ 𝑞)
T T 5. (𝑝 → 𝑞) → (~𝑞 → ~𝑝 )
T F
𝑝 𝑞 ~𝑞 ~𝑝 𝑝→𝑞 ~𝑞 → ~𝑝 (𝑝 → 𝑞) → (~𝑞 → ~𝑝 )
F T
F F T T
T F
F T
2. 𝑝 ∧ ~𝑝 F F
𝑝 ~𝑝 𝑝 ∧ ~𝑝
T
T
F
F

3. ~𝑝 → 𝑞
𝑝 𝑞 ~𝑝 ~𝑝 → 𝑞
T T
T F
F T
F F

40
ASIAN INSTITUTE OF COMPUTER STUDIES (AICS)

CHAPTER 3. LOGIC KEY QUESTIONS


2ND QUARTER


MODULE 12

At the end of this module, you are expected to: How does an argument
WEEK _____

become a fallacy?
 Determine the validity of categorical syllogisms,
 Are all valid arguments
 Familiarize the rules of inference, and
considered as sound
 Establish the validity and falsity of real-life arguments using logical propositions, syllogisms, and
arguments?
fallacies.

3.1.5 Valid Arguments and Fallacies is a tautology, 𝑞 must be true whenever 𝑝1 ∧ 𝑝2 ∧ … ∧ 𝑝𝑛 is true, which means that
𝑝1 , 𝑝2 , … , 𝑝𝑛 are all true. Thus, the validity of the argument [𝑝1 ∧ 𝑝2 ∧ … ∧ 𝑝𝑛 ] → 𝑞
An argument is a compound proposition of the form is equivalent to the proposition [𝑝1 ∧ 𝑝2 ∧ … ∧ 𝑝𝑛 ] → 𝑞 being a tautology.
(𝑝1 ∧ 𝑝2 ∧ … ∧ 𝑝𝑛 ) → 𝑞.
The propositions 𝑝1 , 𝑝2 , 𝑝3 , … , 𝑝𝑛 are the premises of the argument, and 𝑞 is the Example 15. Consider the following argument:
conclusion. Arguments can be written in propositional form, as in above, or in Passing all your courses is sufficient for your graduation.
column or standard form: You passed all your courses.
𝑝1 Therefore, you will graduate.
𝑝2 Show that this a valid form of argument.
⋮ Solution. Let propositions 𝑝 and 𝑞 be:
𝑝𝑛 𝑝: You pass all your courses,
_____ and
∴𝑞 𝑞: You will graduate.

Then the first premise of the argument is 𝑝 → 𝑞. In symbols, we have


The symbol ∴ indicates the conclusion and is commonly read as “therefore”. 𝑝→𝑞
We say that an argument is valid if and only if it is impossible for the conclusion to 𝑝
be false while the premises are all true. This means that in a valid argument when _____
∴𝑞
premises 𝑝1 , 𝑝2 , … , 𝑝3 are all true (consequently, the conjunction 𝑝1 ∧ 𝑝2 ∧ … ∧ 𝑝𝑛
Note that to show that this is valid is equivalent to showing that the proposition
is true), then the conclusion 𝑞 must also be true. In other words, the implication
[𝑝 ∧ (𝑝 → 𝑞)] → 𝑞 is a tautology. Table 3.h precisely shows this since its last
[𝑝1 ∧ 𝑝2 ∧ … ∧ 𝑝𝑛 ] → 𝑞 is always true and so must be a tautology. Conversely, the
column entirely consists of 𝑇’s. Thus, the given argument is valid.
proposition

[𝑝1 ∧ 𝑝2 ∧ … ∧ 𝑝𝑛 ] → 𝑞

41
ASIAN INSTITUTE OF COMPUTER STUDIES (AICS)

𝑝 𝑞 𝑝→𝑞 𝑝 ∧ (𝑝 → 𝑞) [𝑝 ∧ (𝑝 → 𝑞)] → 𝑞 Theorem (Rule of Inference)


T T T T T Let 𝑝, 𝑞, and 𝑟 be propositions.
T F F F T
F T T F T Propositional Form Standard Form
Rule of (𝑝 ∧ 𝑞) → 𝑝 𝑝∧𝑞
F F T F T ∴
Table 3.h Simplification 𝑝
Rule of Addition 𝑝 → (𝑝 ∨ 𝑞) 𝑝

Example 16. Consider the following arguments: 𝑝∨𝑞
Rule of (𝑝 ∧ 𝑞) → (𝑝 ∧ 𝑞) 𝑝
Argument A:
Conjunction 𝑞
If my alarm sounds, then I will wake up. ∴
𝑝∧𝑞
My alarm sounded. Modus Ponens [(𝑝 → 𝑞) ∧ 𝑝] → 𝑞 𝑝→𝑞
Therefore, I woke up. 𝑝

Argument B: 𝑞
If there is a limited freshwater supply, then we should conserve water. Modus Tollens [(𝑝 → 𝑞) ∧ (~𝑞)] → (~𝑝) 𝑝→𝑞
~𝑞
There is a limited freshwater supply. ∴
~𝑝
Therefore, we should conserve water. 𝑝→𝑞
Law of Syllogism [(𝑝 → 𝑞) ∧ (𝑞 → 𝑟)] → (𝑝 → 𝑟)
Argument C: 𝑞→𝑟
If General Antonio Luna is a national hero, then he died at the hands of the ∴
𝑝→𝑟
Americans in 1899. Rule of [(𝑝 ∨ 𝑞) ∧ (~𝑝)] → 𝑞 𝑝∨𝑞
General Luna is a national hero. Disjunctive ~𝑝

Therefore, General Luna died at the hands of the Americans in 1899. Syllogism 𝑞
Rule of [(~𝑝) → ∅] → 𝑝 (~𝑝) → ∅

Note, they are all in the form [(𝑝 → 𝑞) ∧ 𝑝] → 𝑞, or in standard form. Contradiction 𝑝
Rule for Proof by [(𝑝 → 𝑟) ∧ (𝑞 → 𝑟)] → [(𝑝 ∨ 𝑞) → 𝑟] 𝑝→𝑟
𝑝→𝑞
Cases 𝑞→𝑟
𝑝 ∴
(𝑝 ∨ 𝑞) → 𝑟
_____
Table 3.i
∴𝑞
Example 17. Consider the propositions
Hence, by Modus Ponens, all three arguments are valid. However, this does not 𝑝: Julius is hungry,
mean that the conclusions are true. Asserting that an argument is valid simply 𝑞: Jasper is sleeping, and
means that the conclusion logically follows from the premises. 𝑟: Diane is dancing

42
ASIAN INSTITUTE OF COMPUTER STUDIES (AICS)

Rule of 𝑝∧𝑞 Julius is hungry and Jasper is sleeping. Solution. Let 𝑝: “Antonio and Jose are friends.” and 𝑞: “Antonio and Jose are

Simplification 𝑝 Therefore, Julius is hungry. Facebook friends.” Then the given argument is in the form
Rule of Addition 𝑝 Julius is hungry. 𝑝→𝑞

𝑝∨𝑞 Therefore, either Julius is hungry or Jasper ~𝑞
is sleeping. _____
Rule of 𝑝 Julius is hungry. ∴ ~𝑝
Conjunction 𝑞 Jasper is sleeping.
∴ This is valid by Modus Tollens.
𝑝∧𝑞 Therefore, Julius is hungry and Jasper is
sleeping. An argument (𝑝1 ∧ 𝑝2 ∧ … ∧ 𝑝𝑛 ) → 𝑞 which is not valid is called a fallacy. In a
Modus Ponens 𝑝→𝑞 Julius is hungry. fallacy, it is possible for the premises 𝑝1 , 𝑝2 , … , 𝑝𝑛 to be true, while the
𝑝 If Julius is hungry, then Jasper is sleeping. conclusion 𝑞 is false. In this case, the conditional (𝑝1 ∧ 𝑝2 ∧ … ∧ 𝑝𝑛 ) → 𝑞 is not

𝑞 Therefore, Jasper is sleeping. a tautology.
Modus Tollens 𝑝→𝑞 If Julius is hungry, then Jasper is sleeping.
~𝑞 Example 19. Prove that the argument [(𝑝 → 𝑞) ∧ 𝑞] → 𝑝 is a fallacy. This is known
∴ Jasper is not sleeping.
~𝑝 Therefore, Julius is not hungry. as the Fallacy of the Converse.
Law of Syllogism 𝑝→𝑞 If Julius hungry, then jasper is sleeping. Solution. We show that [(𝑝 → 𝑞) ∧ 𝑞] → 𝑝 is a fallacy by means of a truth table.
𝑞→𝑟 If Jasper is sleeping, then Diane is dancing.

𝑝→𝑟 Therefore, if Julius is hungry, then Diane is 𝑝 𝑞 𝑝→𝑞 (𝑝 → 𝑞) ∧ 𝑞 [(𝑝 → 𝑞) ∧ 𝑞] → 𝑝
dancing. T T T T T
Rule of 𝑝∨𝑞 Either Julius is hungry or Jasper is T F F F T
Disjunctive ~𝑝 sleeping. F T T F F

Syllogism 𝑞 Julius is not hungry. F F T T T
Therefore, Jasper is sleeping.
Rule of (~𝑝) → ∅ If Julius is not hungry, then 2 > 10. Since [(𝑝 → 𝑞) ∧ 𝑞] → 𝑝 is not a tautology, the argument is invalid and is hence a

Contradiction 𝑝 Therefore, Julius is hungry. fallacy.
Rule for Proof by 𝑝→𝑟 If Julius is hungry, then Diane is dancing.
Cases 𝑞→𝑟 If Jasper is sleeping, then Diane is dancing. Note that it is sufficient to find the truth values of 𝑝 and 𝑞 that will make the

(𝑝 ∨ 𝑞) → 𝑟 Therefore, Diane is dancing if either Julius premises simultaneously true but the conclusion false. We call such set of values a
is hungry or Jasper is sleeping. counterexample. In this case, the counterexample is the case when 𝑝 is false and
𝑞 is true (see the third row of the truth table above).
Example 18. Determine whether the following argument is valid.
Example 19. Show that the following arguments are fallacies.
If Antonio and Jose are friends, then they are Facebook friends.
Antonio and Jose are not Facebook friends. 𝐴′: If my alarm sounds, then I will wake up.
Therefore, they are not friends. I woke up.

43
ASIAN INSTITUTE OF COMPUTER STUDIES (AICS)

Therefore, my alarm sounded. Example 20. Determine whether the given is a valid argument or a fallacy.
a. Either Aubrey writes a song or plays an instrument with Joy.
𝐵′: If there is a limited supply of freshwater, then I will conserve water.
Aubrey wrote a song with Joy.
I will conserve water.
Therefore, Aubrey did not play an instrument with Joy.
Therefore, there is limited supply of freshwater.
b. Either Aubrey writes a song or plays an instrument with Joy.
Solution. Each of the arguments has the form Aubrey did not play an instrument with Joy.
Therefore, Aubrey wrote a song with Joy.
𝑝→𝑞 c. It is not true that Aubrey writes a song and plays an instrument with Joy.
𝑞
Aubrey did not write a song with Joy.
_____
Therefore, Aubrey played an instrument with Joy.
∴𝑝
Solution. Let 𝑝: “Aubrey writes a song with Joy.” and 𝑞: “Aubrey plays an
Similar to the previous example, each converse is a fallacy of the converse. instrument with Joy.”
The following table lists some of the common fallacies in logic. a. The given argument is of the form
𝑝∨𝑞
Propositional Form Standard Form 𝑝
Fallacy of the [(𝑝 → 𝑞) ∧ 𝑞] → 𝑝 𝑝→𝑞 ∴ ~𝑞
Converse 𝑞
∴ This is the fallacy of Affirming the Disjunct.
𝑝
Fallacy of the [(𝑝 → 𝑞) ∧ (~𝑝)] → (~𝑞) 𝑝→𝑞
Alternate Solution: We can prove that the argument is not valid by finding a
Inverse ~𝑝
∴ counterexample (i.e., truth values for 𝑝 and 𝑞 that make the propositional form of
~𝑞
Affirming the [(𝑝 ∨ 𝑞) ∧ 𝑝] → (~𝑞) 𝑝∨𝑞 the tautology false). This happens when both 𝑝 and 𝑞 are false, as the following
Disjunct 𝑝 table shows:

~𝑞
𝑝→𝑞 𝑝 𝑞 ~𝑞 𝑝∨𝑞 (𝑝 ∨ 𝑞) ∧ 𝑝 [(𝑝 ∨ 𝑞) ∧ 𝑝] → ~𝑞
Fallacy of the (𝑝 → 𝑞) → (𝑞 → 𝑝) ∴
𝑞→𝑝 T T F T T F
Consequent
Denying a [~(𝑝 ∧ 𝑞) ∧ (~𝑝)] → 𝑞 ~(𝑝 ∧ 𝑞)
b. The given argument is in the form
Conjunct ~𝑝
∴ 𝑝∨𝑞
𝑞 ~𝑞
Improper (𝑝 → 𝑞) → [(~𝑝) → (~𝑞)] 𝑝→𝑞
∴ ∴𝑝
Transposition (~𝑝) → (~𝑞)
The first premise can be written as 𝑞 ∨ 𝑝, by the Commutative Law, and so
Table 3.j
we can write

44
ASIAN INSTITUTE OF COMPUTER STUDIES (AICS)

𝑞∨𝑝 Solution. By Modus Ponens, both arguments are valid. We then check soundness
~𝑞 through truth condition.
∴𝑝 a. Note that being poor does not prevent one from serving one’s country (you
Which adheres to the Rule of Disjunctive Syllogism, the argument is valid. can probably think of some examples). Hence, the given argument is a bad
c. In symbols, the argument is in the form argument.
~(𝑝 ∨ 𝑞)
b. It is accepted as true that if one studies every day, then a good work ethic
~𝑝
will be developed. However, it cannot be assumed true that “I study every
∴𝑞
day”. If it is true, then the argument is sound. Otherwise, the argument is
This is the fallacy of Denying a Conjunct.
bad.
Alternate Solution: We can find a counterexample to show that the argument is not
3.1.7 Methods of Proof and Disproof
valid. If both 𝑝 and 𝑞 are false, then the propositional form of the argument is false.

𝑝 ~𝑝 𝑞 (𝑝 ∧ 𝑞) ~(𝑝 ∧ 𝑞) ~(𝑝 ∧ 𝑞)~𝑝 [~(𝑝 ∧ 𝑞)~𝑝] → 𝑞 The main goal of the proof (or proving) is to show that the conclusion logically
F T F F T T F follows from the given proposition (or premises). As for the content of the proof,
they must be based on a given statement (i.e., premise), or they must follow from
the premise via logical equivalences or rules of inferences.
3.1.6 Valid and Sound Arguments
Example 22. Prove the validity of the following argument:
𝑝 → (𝑟 ∧ 𝑠)
A valid argument may still have a false conclusion. When we construct our ~𝑟
arguments, we must aim to construct one that is only valid, but sound. A sound ∴ ~𝑝
argument is one that is not only valid, but begins with premises that are actually
Solution. Thinking process: We assume that all propositions over the line are true
true. It satisfies the truth condition. An argument that does not satisfy the either
(i.e., 𝑝 → (𝑟 ∧ 𝑠) and ~𝑟). From these two propositions, the goal is to establish a
the validity condition of the truth condition is called a bad argument.
logical sequence of propositions to arrive at the conclusion ~𝑝.
Example 21. Determine whether each of the following arguments is valid, and if
A common strategy is to start with the statement not involving the conditional (i.e.,
each is sound.
start with ~𝑟). Now think if ~𝑟 is true, how can we reach ~𝑝?
a. If I was born poor, then I cannot serve my country.
I was born poor. To do that, we use the Modus Tollens on 𝑝 → (𝑟 ∧ 𝑠), but first we need to establish
Therefore, I cannot serve my country. that ~(𝑟 ∧ 𝑠) is true. Since ~𝑟 is true, then by the Rule of Addition, (~𝑟) ∨ (~𝑠) is
b. If I study every day, then I will develop a good work ethic. true. It follows (~𝑟) ∨ (~𝑠) ⇔ ~(𝑟 ∧ 𝑠), by De Morgan’s Law (look on table 3.g
I study every day. and 3.i for reference).
Therefore, I will develop a good work ethic.

45
ASIAN INSTITUTE OF COMPUTER STUDIES (AICS)

The actual proof is written as follows: Proposition Reason


1 𝑝∨𝑟 Premise
Proposition Reason 2 ~𝑝 → 𝑟 Switcheroo
1 ~𝑟 Premise 3 ~(𝑟) ∨ (~𝑠) Premise
2 (~𝑟) ∨ (~𝑠) (1), Rule of Addition 4 𝑟 → ~𝑠 Switcheroo
3 ~(𝑟 ∧ 𝑠) (2), De Morgan’s Law 5 ~𝑝 → ~𝑠 Law of Syllogism
4 𝑝 → (𝑟 ∧ 𝑠) Premise 6 𝑠 Premise
5 ~𝑝 (3). (4) Modus Tollens 7 ~(~𝑠) (6), Double Negation
8 ~(~𝑝) (5),(7), Modus Tollens
Example 23. Prove the validity of the following argument
9 𝑝 Double Negation
𝑝∨𝑟
(~𝑟) ∨ (~𝑠) Example 24. Show that the following argument is invalid: “I would like a career in
𝑠 either teaching or diplomacy. If I teach, then I would want to study abroad.
∴𝑝 Therefore, if I would like a career in diplomacy, then I will study abroad,”
Solution. We can start with the simple proposition 𝑠. Then ~𝑠 must be false since Solution. We first write the argument in symbolic form using the following
𝑠 is taken to be true. By Disjunctive Syllogism (~𝑟) ∨ (~𝑠), it follows that ~𝑟 is propositions:
true. Applying Disjunctive Syllogism again with 𝑝 ∨ 𝑟, it follows then that 𝑝 is true. 𝑡: I would like a career in teaching.
The actual proof is written below: 𝑑: I would like a career in diplomacy.
𝑠: I would want to study abroad.
Proposition Reason Thus, the argument can be written in standard form as
1 𝑠 Premise 𝑡∨𝑑
2 ~(~𝑠) Double Negation 𝑡→𝑠
3 ~(𝑟) ∨ (~𝑠) Premise ∴𝑑→𝑠
4 ~𝑟 (2),(3), Disjunctive Syllogism To show that an argument is not valid, we need to find the truth values for each
5 𝑝∨𝑟 Premise proposition such that the premises are true, but the conclusion is false.
6 𝑝 (4),(5), Disjunctive Syllogism
For 𝑑 → 𝑠 to be false, then 𝑑 must be true and 𝑠 must be false. If 𝑡 is false, then
Alternatively, it is also valid to transform the premises 𝑝 ∨ 𝑟 and (~𝑟) ∨ (~𝑠) to 𝑡 ∨ 𝑑 is true and 𝑡 → 𝑠 are both true. Since there is such a combination of truth
~𝑝 → 𝑟 and 𝑟 → ~𝑠, respectively, using the Switcheroo Law. Then we can use values for 𝑡, 𝑠, and 𝑑 that makes the conclusion false but the premises true, the
Modus Tollens and the Law of Syllogism, as shown below. argument is invalid.

46
ASIAN INSTITUTE OF COMPUTER STUDIES (AICS)

This is an example illustrating that producing a counterexample is sufficient to We assume that the conclusion is false, while the premises are true, and show
show than an argument is invalid. that these lead to a contradiction.

Another method is through the use of an indirect proof or a proof by contradiction. Suppose 𝑝 (the conclusion) is false. Based on the premise, ~𝑞 is true, and so 𝑞
In these proofs, we show that the assumption that the premises are true but the is false. Therefore, 𝑝 ∨ 𝑞 is false, which is a contradiction of the premise that
conclusion is false leads to a contradiction. 𝑝 ∨ 𝑞 is true.

Example 24. Prove the following argument using three methods: (a) via the rules Therefore, the conclusion must be true.
of inference, (b) via truth tables, and (c) via an indirect proof.
We now apply the rules of logic to prove basic results in mathematics. Before
𝑝∨𝑞 we proceed, we state the following important idea:
~𝑞
∴𝑝 Definition in mathematics are always “if and only if” statements.
Consider the following examples:
Solution.
Definition “If and only if” Form
(a) via Rules of Inference An even number 𝑚 is a number that A number 𝑚 is even if and only if it
Proposition Reason can be written as 𝑚 = 2𝑘, where 𝑘 can be written as 𝑚 = 2𝑘, where 𝑘
1 𝑝∨𝑞 Premise is an integer. is an integer.
2 ~𝑞 Premise A binomial is a polynomial with A polynomial is a binomial if and only
exactly 2 terms. if it has exactly 2 terms.
3 𝑝 (1),(2), Disjunctive Syllogism
A parallelogram is a quadrilateral A quadrilateral is a parallelogram if
with two pairs of parallel sides. and only if it has two pairs of parallel
(b) via truth tables sides.

𝑝 𝑞 𝑝∨𝑞 ~𝑞 [(𝑝 ∨ 𝑞) ∧ (~𝑞)] [(𝑝 ∨ 𝑞) ∧ (~𝑞)] → 𝑝


T T T F F T REFERENCES
T F T T T T
Books
F T T F F T
Department of Education, (2016) General Mathematics Learner’s Material
F F F T F T
First Edition 2016, Lesson 36-41, pp. 244-295
Since [(𝑝 ∨ 𝑞) ∧ (~𝑞)] → 𝑝 is a tautology, the argument is valid. Vibal Inc., Chan, J.H.N.,

(c) via indirect proof Miro, E.D., Quiming, R.S., (2016) General Mathematics; Lesson 3.1
Propositions and Connectives; Lesson 3.2 More on Conditionals and Truth
Tables; 3.3 Rules of Inference, pp. 158-183

47
ASIAN INSTITUTE OF COMPUTER STUDIES (AICS)

Electronic Sources

Sundstrom, T. Updated Jun 6, 2019: Statements and Logical Operators


retrieved from
https://math.libretexts.org/Bookshelves/Mathematical_Logic_and_Proof
/

Internet Encyclopedia of Philosophy and its Authors – Validity and


Soundness retrieved from:
https://www.iep.utm.edu/val-snd/

Logical Operators
https://images.app.goo.gl/AfufSUy7phxihc1f6

Waner, S. Costenoble, Steven R. Introduction to Logic (3. The Conditional


and the Biconditional) retrieved from
https://www.zweigmedia.com/RealWorld/logic/logic3.html

48
ASIAN INSTITUTE OF COMPUTER STUDIES (AICS)

a. It is possible for the conclusion to be false while the premises are all
Activity 1: Choose Wisely true.
b. It is impossible for the conclusion to be false while the premises are all
true.
Directions. Encircle the letter of the correct answer. Use black/blue pen only. Any c. It is impossible for the conclusion to be true while the premises are all
form of cheating is prohibited. Submit on the scheduled date given by the true.
instructor.
6. An argument wherein it is possible for the premises to be true while the
1. The symbol ∴ indicates the conclusion and is commonly read as:
conclusion is false.
a. Therefore
a. Valid Argument
b. Before
b. Fallacy
c. Equal to
c. Logical Equivalences

2. An argument that does not satisfy the either the validity condition of the
7. Given 𝑝: “Nairobi eats lunch with Tokyo.” and 𝑞: “Nairobi dances with
truth condition is called a _________.
Tokyo,”
a. Sound Argument
Either Nairobi eats lunch with Tokyo or Nairobi dances with Tokyo.
b. Valid Argument
Nairobi ate lunch with Tokyo.
c. Bad Argument
Therefore, Nairobi did not dance with Tokyo.

3. A _________ is one that is not only valid, but begins with premises that are
The argument given above is a fallacy of:
actually true.
a. Fallacy of a Converse
a. Sound Argument
b. Denying a Conjunct
b. Valid Argument
c. Affirming the Disjunct
c. Bad Argument
8. Via Modus Tollens, the conclusion of the argument “If the secretary resigns
4. A set wherein the truth values of a certain premises are simultaneously
this month, then the manager will shift to another area. The manager did
true, but the conclusion is false.
not shift to another area” is:
a. Co-example
a. Therefore, the secretary resigned this month.
b. Counterpart
b. Therefore, the secretary did not resign this month.
c. Counterexample
c. Therefore, the secretary is also shifted to another area.

5. We say that an argument is valid if and only if _________.

49
ASIAN INSTITUTE OF COMPUTER STUDIES (AICS)

9. “Either Berlin is fooling around, or the Professor is planning seriously. Rule of 𝑝 3.


Berlin is not fooling around. Conjunction 𝑞

Therefore, the Professor is planning seriously.” 𝑝∧𝑞
Modus Ponens 𝑝→𝑞 4.
𝑝
The argument above is valid by the _______. ∴
𝑞
a. Rule of Disjunctive Syllogism Modus Tollens 𝑝→𝑞 5.
b. Rule of Contradiction ~𝑞

c. Rule for Proof by Cases ~𝑝
Law of Syllogism 𝑝→𝑞 6.
𝑞→𝑟
10. A: An argument can only be proved via rules of inference. ∴
𝑝→𝑟
B: A valid argument is also a sound argument. Rule of 𝑝∨𝑞 7.
Disjunctive ~𝑝

With the given statements A and B above, it can be said that: Syllogism 𝑞
a. Both statements are true. Rule of (~𝑝) → ∅ 8.

b. Both statements are false. Contradiction 𝑝
c. Statement A is true, while statement B is false. Rule for Proof by 𝑝→𝑟 9.
Cases 𝑞→𝑟

(𝑝 ∨ 𝑞) → 𝑟

Activity 2: Following by the Rules

Directions. Considering the given propositions, complete the table of rules of


inference below. The first one is done for you.
p: Sheldon thinks like a scientist,
q: Missy plays baseball, and
r: Georgie is the businessman of the family

Rule of 𝑝∧𝑞 1. Sheldon thinks like a scientist and



Simplification 𝑝 Missy plays baseball. Therefore,
Sheldon thinks like a scientist.
Rule of Addition 𝑝 2.

𝑝∨𝑞

50
ASIAN INSTITUTE OF COMPUTER STUDIES (AICS)

GENERAL MATHEMATICS PROJECT A video of them reading their poems at the 2020 Joint Mathematics Meeting in
Denver can be accessed here:
“Mathematics Spoken Word Poetry” https://www.youtube.com/watch?v=VleST_9Ydqo&feature=emb_title
Below is a poster image of the winners of the 2020 AMS Math Poetry Contest (in
Colorado) at the 2020 Joint Mathematics Meeting in Denver. Now, this will be your instructions for the project:

1. This is an individual project. The student will be submitting:


a. A written/printed copy of the student’s poem with mathematics as the
subject
b. Video of student reading their poem
2. The poem should be originally written and formulated by the student. It
should at least have 5 stanzas with 4 verses each stanza. It can be a rhyme
scheme or a free verse, as long as the subject is about mathematics, or any
topics related with the subject. (i.e., you can write about how you feel with
the subject mathematics, or use mathematical terms to describe a moment
you love or hate; you can refer to the poster on the left).
3. After writing a poem, the student should record a video of himself/herself
reading/reciting it. The student can put effort on the clothes he/she is
wearing on the video, and can even put effects on the video itself (i.e.
background song, backdrop, etc.) A maximum of 3 minutes is the time
frame allowed.
4. Your video must be submitted in one of the following file formats: .mov,
.mv4, .mp4, .wmv or any playable format you have on your device. Note
that these are rendered movies, that is, files that will play on someone
else’s computer. Be sure to test your finished product ahead of the
deadline. You may submit them via email, or via any social media platform
chosen by the instructor.
5. Any forms of cheating are prohibited and punishable by 3 - 5 days
suspension and invalidation of your final exam results and project grades
as stipulated in the Senior High School Policies and Standard Operating
Procedures for Students. A committee or the subject instructor himself

51
ASIAN INSTITUTE OF COMPUTER STUDIES (AICS)

should investigate and deal with the said misdemeanor accordingly. Topic/Theme Appropriately Focused topic A vague Lack of focus
focused topic which partially sense of the or confused
with a clear demonstrated purpose of purpose,
Your output will be graded based on this rubric: understanding understanding the topic which results
CRITERIA 5 4 3 2 Score of the intent of of the intent of requiring the in confusion
Delivery A combination A combination Inconsistent Lack of eye the topic. the topic. audience to for the
of appropriate of appropriate use of eye contact, make audience.
and effective eye contact, contact, clarity and assumptions.
eye contact, clarity, and clarity and projection of Total Score:
posture, clarity projection of projection of voice, tone
of expression, voice, tone and voice, tone and pace
projection of pace are used and pace make the SUGGESTED COMPUTATION:
voice, tone and but without a interrupt the reading Suppose a student got the following score:
pacing smooth flow of the difficulty to A. Delivery 4/5
significantly cohesiveness. reading. follow.
enhance the
B. Preparedness 4/5
speakers’ C. Time 5/5
words. D. Enthusiasm/ Creativity in Presenting 3/5
Preparedness Completely Prepared but Seems Does not
E. Topic/Theme 5/5
prepared, could have used somewhat seem at all
memorized more practice prepared, but prepared for Total Score: 21/25
well, and has and tips from the presentation. 21 ÷ 25 = 0.84
obviously used the guest information 0.84 × 100
the material speaker presented
presented by has not been = 𝟖𝟒
the guest practiced
speaker appropriately.
Time Student is Student is Student is Student is
within the time slightly over one minute beyond one
frame allowed time. over time. minute over
(3 minutes max) time.
Enthusiasm / Facial Facial Facial Very little
Creativity in expressions, expressions, expressions, use of facial
presenting ideas and body ideas and body ideas and expressions,
language language body original ideas
generate a sometimes language are or body
strong interest generate a used to try to language;
and enthusiasm strong interest generate does not
about the topic. and enthusiasm enthusiasm generate
It is a creative about the topic. but seem much
presentation. somewhat interest in
lacking. topic

52

You might also like